Estate Planning

अब Quizwiz के साथ अपने होमवर्क और परीक्षाओं को एस करें!

Gerald Carter owns an asset which is worth $100,000 today. It is anticipated that the asset will appreciate at the rate of 10% per year, i.e., in 10 years it will be worth $260,000. If the property is given away now, the gift tax is computed on the $______ (less annual exclusion if allowable).

$100,000 $100,000 is the correct answer. If the asset is not given away and it becomes part of the estate (ten years from today), the estate tax is computed on approximately $260,000. Thus, a gift made currently removes future appreciation from the estate. EGTRRA 2001 repeals the estate tax for one year in 2010.

Dorothy makes a gift worth $23,000 to her son Sam from her estate and makes a gift splitting election. During gift tax computation for her husband, he will be treated as though he has given:

$11,500 If Dorothy, a married individual in a common-law state, gives a gift worth $23,000 to her son and makes a gift splitting election, then while computing the gift tax her husband will be treated as though he gave $11,500 even if the gift did not belong to his property.

Megan and her husband gave each of their three children $30,000 for Christmas. Assuming gift splitting and a $13,000 annual gift exclusion, how much of their combined gift is taxable?

$12,000 Each spouse has made a taxable gift of $6,000 for a combined taxable gift of $12,000. Calculation: Gift: $30,000 x 3 children = $90,000. Gift-splitting: $90,000 divided by 2 = $45,000. Annual Exclusions: $13,000 x 3 = $39,000. $45,000 minus $39,000 = $6,000 in taxable gifts for each spouse, or the couple made $12,000 in taxable gifts this year. Each spouse's taxable gifts of $6,000 are offset by their unified credit of 1,772,800 so gift taxes do not need to be paid.

Let's assume the donor transfers a $100,000 building subject to a $40,000 mortgage on which he or she is personally liable. The donor's creditors collect the $40,000 by proceeding against the pledged building and the donee is subrogated to that creditor's rights against the donor-debtor. Effectively, the donee now stands in the shoes of the creditor. How much more can the donee collect from the donor?

$40,000 The donee can collect an additional $40,000 from the donor.

A wife bought stock ten years ago for $10,000 which is now worth $100,000. She gifts her husband half in a tenancy by the entirety account and dies four months later. The husband sells the stock at her death for $100,000. What amount is subject to capital gains tax?

$45,000 When the husband received the gift of stock, his basis was deemed $5,000. He inherited his wife's stock with a stepped-up basis of $50,000. His new basis is $55,000, therefore $45,000 is subject to capital gains taxes on the sale of the stock.

Assume the estate tax exemption in 2003 is $1 million and the gift tax exemption is $1,200 per person per year. If Sean's gross estate is $1,500,000 and his lifetime non-tax-exempt gifts amount is $30,000, how much of his estate is taxable?

$470,000 $1,500,000 - $1,000,000 - $30,000 = $470,000

Rob and Mary, who are cousins, bought property together two years ago for $80,000. Rob paid $60,000 and Mary paid $20,000. When Rob died last month, the FMV of the property was worth $120,000. What amount was included in his gross estate?

$90,000 $90,000 is included in his gross estate based on the fractional interest rule. Since Rob contributed ¾ of the original purchase price, then ¾ of the FMV of the property, or $90,000 is included in his gross estate.

Gary McNeil is an unmarried individual who made five gifts in 2012 for a total amount of $53,000. In March 2012, he gave cash gifts of $12,000, $8,000 and $3,000 to his father, brother and sister, respectively. In November 2012, he gifted $15,000 each to his father and sister. What is the total amount of annual exclusions that Gary used in 2012?

- $34,000 The annual exclusion allows a donor to make tax-free present interest gifts of up to $13,000 to any number of donees in 2012. Note that this is per donee and not per gift. Therefore, Gary can take a $13,000 annual exclusion for the gifts he made to his father totaling $27,000. He can take an annual exclusion of $13,000 for the gifts he made to his sister totaling $18,000. He can also take an annual exclusion of $8,000 for the gift he made to his brother. The annual exclusions equal $13,000 + $13,000 + $8,000 for a total of $34,000.

There are several gift tax implications when applying federal and state gift tax regulations. Choose the correct tax implications of gifting.

- A gift will remove future appreciation in the property's value from an individual's estate. - A donor must pay a gift tax if the value of the gift to a non-spouse exceeds the annual exclusion and the applicable credit amount. A gift will remove the value of the property and any future appreciation from an individual's estate. Gift tax must be paid if the value of the gift exceeds $5,133,000 after applying the annual exclusion and applicable credit amounts. Taxable gifts are gifts that exceed the gift splitting, annual exclusion and marital and charitable deduction amounts, and are reported on the estate tax return as adjusted taxable gifts.

What is a trustee?

- A trustee is a fiduciary of the trust beneficiary. A fiduciary is legally bound to act, within the confines of the law, in the best interests of the beneficiary. A trustee is in a special position of confidence in relation to the beneficiary because the trustee has control of property that is essentially owned by the beneficiary.

What are the characteristics of a testamentary trust?

- Activated once probate is completed

What are the characteristics of a will?

- Activated once the person dies

The gift tax is tax levied not directly on the subject of the gift itself or on the right to receive the property, but rather on the right of an individual to transfer money or other property to another. In that sense it is:

- An excise tax The gift tax is an excise tax, a tax levied not directly on the subject of the gift itself or on the right to receive the property, but rather on the right of an individual to transfer money or other property to another.

To which of the following transactions would the gift tax not be applicable?

- Assignment of the right to next year's rent from a building to a son - Transfer of property as per divorce settlement enforcement - Tuition fees paid for undergraduate course of a daughter - Medical care payment for a parent Assignment of income creates a situation where gift tax law might also recognize the transfer of a property right, and the present value of a donor's future income could be subject to the gift tax. A qualified disclaimer and payment of medical care are some of the few types of gratuitous transfers that are statutorily exempted from the gift tax. Certain transfers of property between spouses in divorce and separation situations are also exempt from gift tax. Tuition paid to an educational institution for the education or training of an individual is exempt from the gift tax regardless of the amount paid or the relationship of the parties.

Harry would like to have his estate go to his wife Dorothy, and then to their children upon her death. He does not want Dorothy to be able to reassign the beneficiaries of the assets in case she remarries. Which of the following trusts would he want to establish?

- Bypass Trust - Nonmarital Trust - Q-TIP A Q-TIP, bypass trust or nonmarital trust does not allow the beneficiary to be changed.

Which of the following may be reasons to review and possibly adjust estate plans?

- Change in estate tax law - Change in size of estate - Birth of children - Change in marital status - Change in state of residency

What is section 2503(b)

- Does not require the distribution of principal when the minor reaches age 21.This type of trust requires that income must be distributed to the child at least once a year. If the child is still very young, the income can be transferred to a custodial bank account. The child could also be allowed to withdraw a sum equal to the annual gift tax exclusion. The child's parent decides whether the child should be allowed to make this withdrawal. This type of trust doesn't require access at the age of 21, and the principal can continue to be held in the trust as long as the parents wish.

What are the characteristics of an inter-vivos trust?

- During the lifetime of the grantor

Which of the following transfers are considered gifts?

- Endorsed share certificates delivered to donee - Delivery of a building's deed to a third party A gift occurs when endorsed certificates are delivered to the donee or his agent or the change in ownership is delivered to the corporation or its transfer agent. Real property is typically transferred by the delivery of an executed deed. If bonds are titled jointly between the purchaser and another, no gift occurs until the other person has cashed in the bond or has the bond reissued in his name only. An author can give his right to future royalties to his daughter. Such a gift is valued according to its present value. Forgiving a debt constitutes a gift only in non-business situations. When a person pays bills or purchases food or clothing for his wife or minor children, he is not making gifts.

What are the characteristics of a Irrevocable Trust?

- Excluded form Grantor estate -No Grantor control - gains and income taxable to trust

T or F Is the goal of estate planning to Allow court to determine appropriate distribution of estate?

- False Although courts attempt to be fair and equitable when distributing an estate, their decisions may or may not agree with the decedent's intentions. If the decedent has not executed a will, the state's laws of intestacy will determine who the heirs of the assets are as well as the amount of their inheritance. The court distribution process can be lengthy and costly for the beneficiaries. One of the objectives of estate planning is to avoid probate court for as much of the estate assets as possible.

Estate planning does not need to begin until the later part of a person's life. State True or False.

- False Although it becomes more compelling later in life, estate planning should begin when a client begins to accumulate assets. For example, a new parent may want to select who will become the guardian for the minor in the event that one or both parents should become incapacitated or die.

A state that is known as a community property state is one in which all property owned by married people in that state is deemed to be community property. State True or False.

- False Just because a state is known as a community property state does not mean that all property owned by married couples in that state is deemed to be community property. It is possible for some to be owned by him, some to be owned by her, some to be owned jointly with rights of survivorship and some to be owned as a community property.

Cecilia owns her home as a single owner. She is also a sole proprietor of a business that defaulted on its debt. What can the creditors do to Cecilia?

- Force her to liquidate the home to pay off the debt - Place a claim on her home If Cecilia defaults on her loans, the creditors may levy or place a lien on property owned by her or property she owns as joint tenants with others.

A gift for the purpose of the gift tax is a transfer of assets from a donor to a donee, without adequate consideration of money or money's worth. It can thus include:

- Forgiveness of personal debt - Forgiveness of interest on an intra family below market loan - Transfer of benefits of an insurance policy - Discounts on diamond purchases In addition to direct transfers, gifts can also take the form of the forgiveness of a debt, foregone interest on an intra-family interest-free or below market loan, the assignment of the benefits of an insurance policy or the transfer of property to a trust. In community property states, one spouse cannot make a gift of the property to the third person, without the consent of the other spouse. Assignment of income or property done only for tax purposes is not considered a gift. Also, discounts and commissions offered as a normal business practice do not fall in the category of gifts

If either spouse acquires the property by gift or inheritance how would this property retain its status?

- If either spouse acquires the property by gift or inheritance, this property retains its status as separate property even throughout the marriage.

What is future interest?

- Is a legal term that includes reversions, remainders, and other interests or estates. In property law and real estate, a future interest is a legal right to property ownership that does not include the right to present possession or enjoyment of the propert

Jean, Janet, Linda and Laurie inherited a house from their sister as tenants in common. Jean passes away a year later. Who will receive Jean's interest in the house?

- Jean's Estate In tenancy in common, the interest of any owner is passed on to the heirs through the estate upon the owner's death. In this case, Jean's interest in the house is passed on to her estate. If the ownership had been a joint tenancy, then Janet, Laurie and Linda would have received Jean's interest upon her death.

What are the characteristics of a Contractual Relationships?

- Life insurance policies, retirement plans, and bank accounts where benefits are assigned directly to beneficiaries

The recipient of the gift must be?

- Must be the spouse of the donor at the time the gift is made.

Individuals give property away during their lifetimes for many reasons. Some of the reasons include:

- Potential reduction of probate and administrative costs - Protection from creditors claims - Privacy offered by lifetime gifts as compared to gifts through testamentary gifting Some reasons for making lifetime gifts are the privacy that would be impossible to obtain through a testamentary gift, potential reduction of probate and administrative costs, protection from creditor claims, the enjoyment of seeing the donee enjoy the gift, the opportunity for the donor to see how well or how poorly the donee manages the business or other property, and to provide for the education, support and financial well-being of the donee. Additionally, any appreciation accruing between the time the gift is made and the date of the donor's death escapes taxation in the donor's estate. However, taxable gifts (those that exceed $12,000) are brought back into the donor's estate tax calculation as an adjusted taxable gift, since gift and estate taxes are progressive and cumulative.

Separate property is:

- Property acquired through gift after marriage - Property acquired prior to marriage by either spouse - Property acquired through inheritance after marriage Property acquired prior to marriage remains separate property, even after marriage. Property acquired as gift or inheritance even after marriage retains the status of separate property. But property acquired by either or both of the spouses themselves is deemed to be community property, unless titled otherwise.

What are the characteristics of a joint tenancy?

- Property automatically passes at death to the surviving co-owner

Which of the following is/are similarities between JTWROS property and tenancy by the entirety property?

- Property passes by operation of law. - Property avoids ancillary probate if located out of state. - Property title is considered a will substitute. - Property may be attached by joint creditors. ------------------------------------------------------ However, unlike a Joint Tenancy with Rights of Survivorship, neither party in a tenancy by the entirety has a unilateral right to sever the tenancy.

What can a couple do to minimize estate costs and taxes?

- Purchase insurance within an irrevocable life insurance trust (ILIT) Estate taxes and costs can be minimized by purchasing a life insurance policy within an irrevocable life insurance trust (ILIT). Revocable trusts are tax-neutral and estate property cannot be declared as gifts. Over-qualifying the estate for the marital deduction would save estate taxes at the first spouse's death, but would be taxed in the surviving spouse's estate.

Which of the following are examples of testamentary trusts that can be established by a will?

- Q-TIP - Bypass - Sprinkle trusts Testamentary trusts are activated upon the completion of a probate process. They may include Q-TIP trusts, bypass trusts and sprinkle trusts. A revocable trust is an example of an inter vivos or living trust, which is intended to provide for the grantor during his or her lifetime.

What is section 2503(c)

- Requires the distribution of principal when the minor reaches age 21.A Section 2503(c) trust allows all the principal and income to be used for the child until he reaches the age of 21, unlike the 2503(b) trust that extends beyond age 21 and requires income to be paid to the child annually. The trustee can pay the child's college expenses from the 2503(c) trust. When the child turns 21, all the money remaining in the trust must be turned over to him, unless he decides to extend the trust. Before choosing this trust, the grantor should consider whether the child would be mature enough at 21 to handle the money responsibly.

In order to minimize estate taxes, what can Sean do with his estate in 2012?

- Spend it all. - Give it away to charity. - Give it all to his spouse. - Transfer an insurance policy to a trust more than 3 years from the date of his death. - Have an irrevocable life insurance trust purchase a new policy.

A well-known novelist contracted with a motion picture company to make a movie on one of his famous works. In return the novelist was to receive a lump sum of $200,000, plus a royalty on the earnings of the movie of 10% of the gross sales. Before the movie was released for public viewing, he sold his son the right to his services for $1. The son, in turn, transferred the rights to the contract to a trust. Assume that in the first month after release, that is December 2011, the movie grossed $1 million. Who was due to pay the taxes and how much is the gross taxable income?

- The novelist must pay tax on a gross taxable income of $300,000. This is a sham gift and so the entire series of transactions had no gift tax effect. The income was completely taxable to the novelist. Taxable income is $200,000 + 0.10 x $1,000,000 = $300,000, on which the novelist is liable to pay income tax.

The requirements of a valid and qualified disclaimer are:

- The refusal must be in writing. - The person disclaiming must not have accepted the interest or any of its benefits. The refusal must be in writing for it to be a valid and qualified disclaimer. The person disclaiming must not have accepted the interest or any of its benefits. The writing must be received by the transferor, his legal representative or the holder of the legal title to the property no later than nine months after the later of the date on which the transfer creating the interest is made, or the date the person disclaiming reaches age 21. Due to the refusal, someone other than the person disclaiming receives the property interest, but the person making the disclaimer cannot in any way influence who is to be the recipient of the disclaimed property.

Which of the following are objectives of estate planning?

- Transfer assets at death consistent with wishes - Transfer assets intact without losing much to taxes Estate planning ensures that the distribution of a person's estate is in accordance with his or her wishes, rather than determined by a court appointed method through the rules of intestacy. It also helps to minimize estate and gift taxes so that the estate is as intact as possible.

John and Elizabeth are married and are residents of Nevada. John is software professional and is the only earning member in the household. They buy property with John's money. After ten years of marriage, John files a divorce. Elizabeth claims and gets a share of the property, after the divorce. State True or False.

- True Nevada, being a community property state, follows the principle that even if only one of the spouses is the income earner, the efforts of the non-working spouse contributed to the purchase of the marital property.

Probate assets are those assets that are individually owned by the decedent at death. State True or False.

- True Probate assets are those assets that are individually owned by the decedent at death, that is, solely owned assets, interest in tenancy-in-common assets, interest in community property, or beneficiary designation assets, such as life insurance, where the owner's "estate" is named as the beneficiary.

John Nash and his wife Susan Nash are joint owners of a property. Susan can terminate the ownership without John' permission. State True or False.

- True The joint property law does not require the consent of all joint tenants before termination. One JT can change property to tenants in common without the other JT's consent. For property owned as JTWROS between spouses, it's a good idea to get the spouse's signature on a deed transfer form to avoid having the JT spouse assert survivorship rights on property sold to a third party.

Husband and wife are co-managers of their community property. If the wife dies and bequeaths her half of the community property to her children, both the husband's and children's property interests will receive a step-up in basis to FMV at the wife's death. State True or False.

- True The wife's one half ownership of community property is included in her estate, and will receive a step-up in basis when the children inherit her share. The husband's one half of the couple's community property will also receive a step-up in basis at his wife's death. -------------------------------------------------------------- when you own a home as community property the surviving spouse will receive a step-up in cost basis on the entire value of the home, not just the 50 percent that was the deceased spouse's share of the asset.

Harry Ford and his wife own property jointly with right of survivorship (JTWROS). His wife transfers her interest in property to him and then he set up JTWROS with his business partner. Is Harry allowed to do this? State True or False.

- True There is a common tendency for people to believe that a joint tenancy with right of survivorship only exists between spouses. But the fact is that it can be stretched to accommodate parent and child, brother and sister, and business partner and business partner.

Mr. and Mrs. Murphy are contemplating purchasing a new home. When presented with property titling options, which of the following statements are true?

- With tenancy by the entirety, they do not have to worry about creditors forcing sale on the house from the debt of either spouse. -The most important difference between a tenancy by the entirety and a joint tenancy or tenancy in common is that a tenant by the entirety may not sell or give away his interest in the property without the consent of the other tenant. Tenancy by the entirety protects the property from forced sales (levy) to compensate for default on a loan by either spouse. If the Murphys bought the house as joint tenants with right of survivorship or as tenants by the entirety, in the event of one owner's death, the property automatically passes to the surviving spouse, not through probate.

Does the right to use property such as money at no charge constitute a gift of property?

- Yes Interest-free and below-market-rate loans are treated as taxable gifts.

What are the characteristics of a revocable trust?

- grantor control - living trust - included in grantor's estate

A well-known golfer contracted with a company to make pictures depicting his form and golf style. In return the golfer was to receive a lump sum of $120,000 plus a 50 percent royalty on the earnings of the picture. But before any pictures were made, he sold his father the right to his services for $1. The father, in turn, transferred the rights to the contract to a trust for his son's three children. The income would be taxable to whom?

- the golfer The court held that the entire series of transactions had no tax effect and that the income was completely taxable to the golfer.

The tax rates are applied to _____ _______ taxable gifts rather than only to taxable gifts made in the current calendar year.

- total lifetime total lifetime taxable gifts. The gift tax is based on the value of the property transferred. It is computed on a progressive schedule based on cumulative lifetime gifts.

Which of the following choices would be included in a broad definition of a gift? Click all that apply.

- transfers of partnership interests - cancellation of a debt - gifts of royalty rights - gifts of checks or notes to third parties

What are three types of property ownership that avoid probate?

1. Assets jointly owned with rights of survivorship 2. Assets transferring under a beneficiary designation 3. Assets owned within a trust prior to death

There are certain valuation problems unique to the gift tax. What are these problems associated with?

1. Indebtedness with respect to transferred property 2. Restrictions on the use or disposition of property 3. Transfers of large blocks of stock 4. Valuation of mutual fund shares 5. Valuation of life insurance and annuity contracts

What percentage of the assets value on the date of death will be included in the owner's gross estate after death?

100% 100% of the assets value on the date of death will be included in the owner's gross estate after death.

According to Internal Revenue Code Section 2040(a), except for joint tenants who are husband and wife, unless the surviving owner can prove contribution toward the purchase of the asset, what percentage value of jointly held property will be included in the gross estate if the first owners die?

100% Whether the ownership share is included in the decedent's gross estate depends on whether: a) it was owned JTWROS with a spouse, or b) it was owned JTWROS with a non-spouse. If the asset is titled JTWROS or tenancy by the entirety with a spouse, then only 50% of the value will be included in the decedent's gross estate. If the asset is titled JTWROS with a non-spouse, then 100% of the value is included in the decedent's gross estate. However, if it can be proven that the other owner(s) actually contributed to the purchase of the property, then only the percentage contributed is included in decedent's gross estate. Read more: http://www.investopedia.com/exam-guide/cfp/property-titling-transfer/cfp4.asp#ixzz3WT4xg1bY Follow us: @Investopedia on Twitter

Assume a donor places $10,000 into a Section 2503(b) trust that is required to pay his 10 year-old daughter all income until she reaches age 25. Assume a 6.0% Section 7520 rate which results in a Table B factor amount of .582735 for 15 years. What is the present value of the income the daughter will receive for the next 15 years?

5,827 - $5,827, that is, $10,000 times 0.582735. If the income were payable for her entire life, the present value would jump to almost $9,621. The donor can take an annual exclusion equal to the PV of the income interest, or $5,827, in the year the trust is created.

Which of the following states has some form of community property law?

Consequently, the term "community property" does not mean exactly the same thing in each of the nine states that have some form of community property laws. Indeed, to this day the courts continue to refine the concept. The states which have some form of community property law are: Arizona, California, Idaho, Louisiana, Nevada, New Mexico, Texas, Washington and Wisconsin.

What is a disclaimer trust?

- used when the surviving spouse wishes to file a qualified disclaimer over a portion of the property passing to the survivor from the grantor's estate.

Which of the following can help reduce estate taxes?

- Irrevocable Trusts The grantor gives up control and right to the assets of an irrevocable trust. Therefore, the assets are not included in the estate taxes.

Which of the following are legal capacity requirements of an executor?

- Age of Majority - Business and investment experience Legal capacity requires that the executor be a U.S. citizen and satisfy state law requirements such as age of majority, mental competency, and domicile.

Laura places two million dollars in an irrevocable trust for the benefit of her son William and the remainder to her grandson George. At William's death in 2012, a taxable termination occurs. Assuming no exemptions are claimed and there's no appreciation in the trust, what is the net amount for George?

$1,300,000 The GST applicable rate is 35% of $2 million which equals $700,000. This amount must be paid out of the $2 million property passing to the grandson George. Therefore, George only nets $1,300,000.

Laura is the surviving spouse of Kevin, who passed away five years ago. As the surviving spouse, she has a 5/5 limited power of invasion over the corpus of the Q-TIP trust left by Kevin. Her annual income is $5,000. The trust corpus amount is $200,000. What amount of corpus can Laura withdraw?

$10,000 The surviving spouse may only withdraw the greater of $5,000 or 5% of the trust corpus annually, if the trust is structured to give the spouse this power of withdrawal. In Laura's case, annual income is $5,000 and 5% of $200,000 is $10,000. Therefore, she can withdraw the greater amount, or $10,000.

Bill's farm had a good harvest in August 2012, bringing in an income of $80,400. The dairy products from the farm resulted in a regular income of $3,000 per month, which was received at the end of each month. Bill died on October 13, 2012. How much of the income will be included in Bill's final income tax return?

$107,400 The income that Bill earned and received has to be included in his final income tax return. This amount is the sum of $80,400 that he received in August before his death and $27,000 ($3,000 x 9 months) that he received on a monthly basis from January through September. The remaining $3,000 monthly payments from October through December would be included in the recipient's taxes.

Roger establishes a CRUT in the year 2012 by placing $200,000 into the corpus of the trust. The terms of the trust specify that Roger will receive 6% of the re-valued assets of the trust annually. If the trust earns $20,000 in 2012, then: 1. How much is distributed to his wife Love, who is the trust beneficiary, in year one of the trust? 2. How much is distributed to Love in year two of the trust?

$12,000 in year one and $12,480 in year two. Once Roger establishes the trust, the corpus must pay out a specified amount of income (a fixed percentage) each year. As the trust is a CRUT, it must pay out the stated percent of the annually reappraised value of the corpus. Therefore, if in year one $200,000 is placed in the corpus of the trust by Roger, and the terms of the trust specify that 6% of the trust assets must be distributed to Love, then $12,000 would be distributed to her. However, as the trust had earned $20,000 in year one, the remaining undistributed $8,000 would be added to the corpus of the trust and in year two, 6% of $208,000, or $12,480, would be distributed to Love.

Assume Susan dies in 2012. Susan's gross estate is valued at $6 million which includes her IRA worth $400,000 that she is leaving to her nephew John. Her net federal estate tax liability is $308,000. Susan's net federal estate tax liability would be $168,000 if she did not include the value of the IRA in her estate. Assuming a lump-sum distribution of the IRA by John, what is the amount of the IRD deduction that John can take to reduce his income tax liability on the IRA he inherited from Susan?

$140,000 To calculate the IRD deduction it is the difference between the net federal estate tax that includes the IRA ($308,000) and the net federal estate tax that would not include the IRA ($168,000) or $140,000.

Let's assume the donor owns a capital asset, which qualifies as a long-term capital asset, for example, corporate stock. The value of the stock on the date of the gift is $30,000. The donor's AGI is $60,000. Since a long-term capital asset is being gifted, what is the maximum charitable deduction the donor can take?

$18,000 Since a long-term capital asset is being gifted, the maximum charitable deduction the donor can take is $18,000 (30% of $60,000).

James and his wife Sally jointly purchase 2,000 shares of Nedloyd Corp. at the rate of $25 per share. They shared the payment of $ 50,000. James contributed $15,000, while Sally contributed $35,000. At the time of James's death the stock is worth $200 per share. How much will be includable in his gross estate?

$200,000 The 50-50 rule applies to joint tenancy between spouses. The 50-50 rule provides that only 50% of the current market value of shares held jointly by James and Sally will be includable in his estate. This is regardless of the size of James's original contribution. In fact, even if Sally can prove that she contributed more towards the whole of the original price of the shares, this rule must be used.

Bob Smith donated his ancestral home to the Carnegie Foundation, a private, nonoperating foundation. He bought it for $60,000 five years ago. The FMV of the property had appreciated to $100,000 when he donated it. His AGI was $120,000. What is the maximum charitable deduction that can be taken in the year of the contribution?

$24,000 The Carnegie Foundation is a private, nonoperating foundation. Therefore, the maximum charitable contribution that can be taken in the year of the contribution is limited to the donor's (Bob's) basis in the property, subject to the 20%/30% AGI rules. Because the Carnegie Foundation is not a public charity, the maximum deduction for a long-term capital asset is 20% of AGI, or $24,000. Since the value of the gift is $60,000, the difference between $60,000 and $24,000 may be carried forward for five years. This is equal to if the property has appreciated in value since the date of its acquisition and the donee is a private nonoperating foundation.

Philip sets up a CRAT, as he wishes to provide his wife Meryl with a stream of income to last for the life of the income recipient. On her death, the remainder interest in the property passes to the Heart Association. The corpus amount is $500,000. He wants to pay Meryl only the least income possible to be paid from the corpus. What is the minimum amount he must pay her?

$25,000 Once the trust is established, the corpus must pay out a specified amount of income (a sum certain) each year. In a CRAT, the trust must pay a minimal amount of 5% of the initial value of the corpus.Therefore, the minimum amount Philip has to pay Meryl is 5% of the corpus amount (that is $25,000).

Russell Patterson and his wife Lydia jointly purchased 5,000 shares of Warren Entertainment Inc. at the rate of $50 per share. They shared the payment of $250,000. Russell contributed $50,000, while Lydia contributed $200,000. At the time of Russell's death in 2012, the stock is worth $100 per share. How much will be includable in his gross estate?

$250,000 The 50-50 rule applies to joint tenancy between spouses. This rule provides that only 50% of the current market value of shares held jointly by Russell and Lydia will be includable in Russell's gross estate when he dies. This is regardless of the size of Russell's original contribution. In fact, even if Lydia can prove that she contributed more towards the whole of the original price of the shares, this rule must be used.

Calculate the total GST tax, assuming no growth, if the value of the transfer is $5 million, the federal and state taxes are $750,000, charitable deductions were $150,000 and $3.5 million GST exemption is available. Assume the maximum GST tax rate is 35%.

$256,025 1. Applicable fraction: 3,500,000 / (5,000,000 - (750,000 + 150,000) = 0.8537 2. Inclusion ratio: 1 - 0.8537 = 0.1463 3. Applicable rate: 0.1463 x 0.35 = 0.0512 4. Total GST: 5,000,000 x 0.0512 = 256,025 For any year in which a gift is given that exceeds the annual exclusion or for which gift splitting was elected, Form 709, United States Gift (and Generation-Skipping Transfer) Tax Return must be filed. The gift tax is calculated by adding the value of all gifts given over a lifetime that is above the exclusion and subtracting deductions and exemptions: •+ Fair Market Value of all gifts for tax year.◦- Marital Deduction ◦- Charitable Deductions ◦- Annual Exclusion Available for each Donee ◦- Unlimited Annual Exclusion for Payments to Providers of Educational or Medical Services for the Donee •= Taxable Gifts for current year. •+ Taxable Gifts from Prior Years.◦Taxable gifts from prior years must be added because the gift tax is cumulative—the tax rate depends on the total value of gifts given over the donor's lifetime. •= Total Gift Value •Tentative Tax = Total Gift Value × Applicable Tax Rate◦- Gift Taxes Paid or Deemed Paid ◦- Unified Tax Credit •= Total Tax Due

William gifted his one-acre farmland (held long-term) to the United Fund. He had bought the farmland back in the 1980s for $10,000. The present FMV of the farmland is $60,000. His AGI is $100,000. What will be his maximum charitable income tax deduction for this tax year?

$30,000 As William's farmland is long-term capital gain property, he is entitled to take a maximum charitable income tax deduction equal to 30% of his AGI, based upon the FMV of the AGI. Therefore, the maximum charitable income tax deduction to be taken for long-term capital gain property is 30% of $100,000, or $30,000. Therefore, if the $60,000 gift were made, only $30,000 would be allowed as a charitable deduction, with the remaining $30,000 deduction carried forward for the next five years.

Gary McNeil is an unmarried individual who made five gifts in 2007 for a total amount of $53,000. In March 2007, he gave cash gifts of $12,000, $8,000 and $3,000 to his father, brother and sister, respectively. In November 2007, he gifted $15,000 each to his father and sister. What is the total amount of annual exclusions that Gary used in 2007?

$32,000 The annual exclusion allows a donor to make tax-free present interest gifts of up to $12,000 to any number of donees in 2007. Note that this is per donee and not per gift. Therefore, Gary can take a $12,000 annual exclusion for the gifts he made to his father totaling $27,000. He can take an annual exclusion of $12,000 for the gifts he made to his sister totaling $18,000. He can also take an annual exclusion of $8,000 for the gift he made to his brother. The annual exclusions equal $12,000 + $12,000 + $8,000 for a total of $32,000.

Callie donates her personal jewelry and two used cars to the Society for the Prevention of Blindness. The Society for the Prevention of Blindness uses the two cars for transportation, but intends to sell the jewelry set and use the sales proceeds. The jewelry cost her about $10,000 two years ago when she bought it for her cousin's wedding. And the two cars were bought five years ago for $20,000 each. The FMV of the two cars, when she donated them to the qualified charity, was $15,000 each. The personal jewelry set's FMV was $15,000. Her AGI is $90,000. What is the total maximum charitable deduction that she can take for the year?

$37,000 For use related items, the maximum charitable deduction is 30% of AGI, that is, 30% of $90,000, or $27,000. Since the value of the cars is $30,000, the $3,000 not utilized as the deduction can be carried forward for the next five years. For non-use related items, the deduction is equal to 50% of AGI, that is, 50% of $90,000, or $45,000. Given that the basis of the jewelry is $10,000, less than the allowable $45,000, the full value of the basis may be taken as the charitable deduction. The deduction is $27,000 plus $10,000, or $37,000.

What is any occupation?

- Insured is totally disabled to perform the major duties of any gainful occupation for which he is reasonably suited.

Benjamin and his son Jerome jointly purchase 2,500 shares of Petra Inc. at the rate of four dollars per share. Benjamin paid $7,500, while Jerome paid the remaining $2,500 required for this purchase. At the time of Benjamin's death in 2012, the stock is worth $20 per share. How much will be includable in his gross estate?

$37,500 The percentage-of-contribution rule will be applied here because the property is held jointly with the right of survivorship. This provision measures the estate tax by referring to the portion of the purchase price attributable to Benjamin's contribution, which was 75%. Therefore, the value includable in his gross estate is 75% of market value of the shares when he dies, that is, 75% of $20 x 2500, which is $37,500.

Jennifer's cousin Ann had a bonus check of $2,000 from her employer that came in the mail after Ann passed away in 2012. The executor included this bonus amount in the value of the estate. If estate tax was 20% of the estate, and Jennifer is the sole beneficiary of the estate, how much will she be able to deduct for IRD income tax purposes?

$400 Jennifer will be able to deduct 20% of the $2,000, or $400, which is subject to both estate tax on Ann's estate and Jennifer's personal income tax.

Harold Free inherited a piece of residential real estate purchased by his uncle for $5,000 and valued in his uncle's estate at $10,000 in 1975. In 1985, Mr. Free transferred the property into joint tenancy with right of survivorship with his girlfriend. The property then had a fair market value of $25,000. Just before Mr. Free died in 1992, his girlfriend put an addition on the property costing $5,000. The property was then valued at $30,000. The girlfriend sold the property in 2002 for $35,000. What is her taxable gain?

$5,000 When Mr. Free died, the entire value of the property was included in his gross estate, except for the addition costing $5,000. The property is included in Free's estate under the fractional interest rule, because Free contributed all of the cost except for the addition. The girlfriend's basis, with the cost of the addition added on, is $30,000, so she has a gain of $5,000 on the sale.

Peter Philips made a lifetime gift of $6,500,000 to his granddaughter, Amanda, this year. If no exemptions were made, then the net amount for Amanda would be:

$6,500,000 A gift from an individual to a grandchild is called a direct skip. In the case of a direct skip the transferor must pay the gift and GST tax. Hence, Peter must pay a generation-skipping transfer tax of $2,275,000 ($6.5 million x .35) in addition to a gift tax that would be paid out of his other assets, not out of his gift to Amanda. So Amanda would net $6,500,000.

Many of Ned Howard's relatives and friends traveled from various parts of the country to be present at Ned's funeral. His family had made it clear to the undertakers that they must spare no effort in caring for the guests. The undertaker's bill consisted of the following particulars: Interment expenses $700. Catering expenses for guests $4,500. Cost of burial lot $3,000. Marble headstone with gold engraving $2,200. Hearse and transportation charges 1,800. In calculating adjusted gross estate, how much is allowable as funeral expenses?

$7,700 Funeral expenses, subject to certain limitations, are deductible. Such expenses would include interment, burial lot or vault, grave marker, perpetual care of the gravesite and transportation of the body to the place of burial. It would not include amount spent on caring for guests.

Dan's gross estate is $2,250,000, administrative and funeral costs are $250,000. To qualify for Sec. 303 redemption, the value of the stock in question must exceed what?

$700,000 35% ($2,250,000 - $250,000) = $700,000.

Stan and his son Bill purchased 200 shares of stock for $100,000. Bill contributed $60,000. At Bill's death, 60% of the value of the stock at that time is includable in Bill's estate. If the stock has an estate tax value of $120,000, how much would be includable in Bill's estate?

$72,000 If the stock has an estate tax value of $120,000, $72,000 (0.60 x $120,000) would be includable.

If Lloyd's AGI is $160,000, and he makes a $90,000 cash contribution to the United Way, what would his charitable deduction be for the year of the gift?

$80,000 If the gift is a gift of cash, the donor is allowed a maximum annual charitable income tax deduction equal to 50% of the donor's AGI. Therefore, as Lloyd's AGI is $160,000, and he makes a cash contribution of $90,000 to a qualified public charity (the United Way), he will be entitled to take a charitable income tax deduction equal to 50%, or $80,000. The remaining $10,000 would be carried over into the next tax year and could be deducted on his income tax return as a charitable deduction for that year. Keep in mind that there is a 5-year carry-forward, not just a one-year carry forward. Therefore, if some or all of the $10,000 cannot be used the following year, the taxpayer has an additional four years to use it.

"A" and "B" are married. They bought a farm in 1995 for $100,000 and titled it in joint names with right of survivorship. Assume that in 2009 the property is worth $250,000 when "A" dies. "B" sells the property in 2012 for $270,000. If "A" and "B" did not use the farm as a residence, what is "B's" taxable gain?

$95,000 When the farm was acquired, the joint tenants had a total basis of $100,000, which was divided between them equally. When "A" died, "A's" one-half interest received a step-up in basis to the date of death value. At "A's" death, the property was worth $250,000, so "A's" one-half interest was worth $125,000. "B" then had a basis of $125,000 in the interest received from "A" and a basis of $50,000 in "B's" original one-half interest. "B's" new basis, therefore, is $175,000. The property is sold for $270,000, so the gain is $270,000 - $175,000 = $95,000.

Which of the following statements concerning the generation-skipping transfer tax are correct? (I) The tax is in addition to any estate or gift tax that would be due on the transfer. (II) The tax equals the highest estate and gift tax rates. (III) The tax is imposed on the beneficiary in a direct-skip, so the residual estate of the transferor is not reduced by the amount of the tax. (IV) If a gift qualifies for the gift tax annual exclusion, it will also qualify for the GSTT annual exclusion.

(I) and (II) only In a direct-skip, the tax is paid by the transferor or by the estate, so the beneficiary receives the full amount of the gift or testamentary transfer, and the residual estate is reduced by the amount of the tax. Outright lifetime gifts that qualify for the gift tax annual exclusion will also qualify for the GSTT annual exclusion. Gifts in trust, however, may qualify for the gift tax annual exclusion and not qualify for the GSTT annual exclusion. In order for the gift in trust to qualify for the GSTT annual exclusion, the trust must provide that it will make no distribution to any person other than the beneficiary during the beneficiary's life. In addition, at the beneficiary's death, the trust assets must be includible in the beneficiary's gross estate.

Which of the following statements concerning a will are correct? (I) A will can be altered, amended, or completely rewritten at any time before a person's death. (II) A will cannot be rewritten but can be amended at any time by a codicil. (III) A will must be signed by the maker and usually must be witnessed by two or three people. (IV) A will assures a testator of orderly disposition of his or her property, because the will is effective as soon as it is signed and witnessed.

(I) and (III) only (I) and (III) only is the answer. A will can be altered, amended, or completely rewritten at any time before a person's death. A will can also be amended at any time by a codicil. A will must be signed by the maker and usually must be witnessed by two or three people. A will assures a testator of the orderly disposition of his or her property, but the will is not effective until the testator is dead.

Alex Fernandes created an irrevocable trust in which he placed securities valued at $1 million. His wife, Jeanette, will receive the income from the trust for her lifetime, and at her death, the trust assets will be divided between their sons Michael and Julien. Which of the following statements concerning the Fernandes trust is (are) correct? (I) Michael has a remainder interest in the trust. (II) Julien has a reversionary interest in the trust. (III) Jeanette has a life interest in the trust. (IV) Alex has a reversionary interest in the trust.

(I) and (III) only Michael has a remainder interest in the trust, because his interest does not become operative until after the termination of his mother's life interest. A reversionary interest is a right the grantor retains to recover the property, so Julien has a remainder interest and not a reversionary interest. Jeanette has a life interest in the trust, because she will receive the income for life. Alex has not retained a reversionary interest in the trust.

Which of the following statements concerning the role of the financial planner in estate planning is (are) correct? (I) The planner can help the client to identify his or her financial problems. (II) The planner may offer a legal opinion as to the reliability of the estate plan. (III) The planner's participation in the estate planning process may range from little involvement to total involvement.

(I) and (III) only The financial planner will frequently be called upon to advise a client about those estate planning objectives that the client is financially incapable of attaining. Identifying these financial constraints and problems is necessary at an early stage in the estate planning process. The financial planner may play a very large role or almost no role in the estate planning process. In any event, the financial planner must not offer a legal opinion concerning the estate plan. Only a lawyer is permitted to give legal opinions.

Which of the following discounts are used to leverage gift tax annual exclusions in family limited partnerships and limited liability companies? (I) Minority discounts. (II) Blockage discounts. (III) Key person discounts. (IV) Marketability discounts. (V) Co-ownership discounts.

(I) and (IV) only Minority and lack of marketability discounts are used for business interests to increase the annual exclusion amounts that can be gifted annually.

Which of the following are among the advantages of the probate process? (I) limited time period for any creditors to file their claims (II) avoidance of income tax while the property is in probate (III) reduction in cost of estate administration

(I) only (I) only is the answer. The probate process affords only a limited time for creditors to file claims against the decedent's estate. Otherwise, claims may be filed for as long as the statute of limitations on suits. Income tax savings would result for the beneficiaries if they are in high tax brackets, but income tax is not completely avoided while the property is in probate. The estate must pay income tax. Estate administration costs are increased by probate.

In which of the following circumstances might a testator be able to disinherit his or her surviving spouse? (I) A valid ante-nuptial agreement was executed by the spouse. (II) A state statute permits the surviving spouse to take a specified percentage of the estate if the percentage of the decedent's estate left to a spouse is less than the statutory share. (III) The spouse was the testator's second wife. (IV) In a community property state, the decedent left a will directing that all of the community property be placed in a trust for the decedent's children.

(I) only A surviving spouse who enters into an ante-nuptial agreement may waive the right to take the elective share of the estate. Otherwise, a surviving spouse retains a right to an elective share when disinherited. In a community property state, a surviving spouse also has a right to an elective share of one-half of community property, but this right may be given up to obtain benefits conferred conditionally by the will.

George Foster has created an irrevocable trust with $2 million worth of rental properties. The trust provides that the income from the assets will be paid annually to George's wife, Margery. At Margery's death, the assets will pass to George's four children according to whatever plan of distribution Margery directs in her will. Which of the following statements concerning the Foster trust is (are) correct? (I) Margery has a limited power of appointment over the trust assets. (II) Margery has a general power of appointment. (III) The four children are the donees of the power of appointment. (IV) George is the holder of the power of appointment.

(I) only Margery has a power of appointment over the trust assets, because she can make disposition of the grantor's (George's) property in her will. Margery has a special power of appointment, because she can appoint the assets only to specified persons, and not to herself or to her estate. Margery is the sole donee or holder of the power of appointment, and the four children are the remaindermen. George is the grantor and not the holder of the power of appointment.

Which of the following statements concerning community property are correct? (I) Property received as an inheritance or gift to one spouse during marriage is separate property. (II) Only one-half of community property is subject to each spouse's will. (III) All community assets receive a step-up in basis following the death of one of the spouses. (IV) Community property is fully includible in the decedent's gross estate.

(I), (II) and (III) only (I), (II) and (III) only is the answer. (IV) is incorrect, because only one-half of community property is includible in the decedent's gross estate. All community assets, however, receive a full step-up in basis following the death of one of the community tenants. Only one-half of community property may be disposed of by Will, and the property may pass to heirs other than a spouse.

A residuary clause is typically included in a will for which of the following reasons? (I) To provide a method for distributing any portion of the remaining estate after payment of all debts, taxes, administrative expenses and specific bequests. (II) To prevent any portion of the remaining estate's assets from being distributed in a manner that is inconsistent with the testator's wishes. (III) To avoid partial intestacy with regard to particular items of property. (IV) To provide for disposition of property that is acquired after execution of the will.

(I), (II), (III) and (IV) A residuary clause bequeaths all property remaining after specific bequests. The debts, taxes, and administrative expenses must be paid before any distribution to legatees. A residuary clause can protect against acquisition of future property and partial intestacy for particular items of property.

Which of the following is (are) valid reasons for avoiding the use of joint and mutual wills? (I) Mutual wills may be unenforceable after the first decedent dies. (II) Joint wills between spouses may result in loss of the marital deduction. (III) Joint wills cannot be changed. (IV) These wills may not result in disposition of property as seems best to a testator under existing circumstances.

(I), (II), (III), and (IV) Joint wills are not recommended because the property interests of the first decedent spouse may be construed as terminal interests, and thus will not qualify for the marital deduction. Whether the survivor may write a new will to change the terms of a joint will, is questionable. A mutual will can be changed. Finally, it is difficult to enforce an agreement for mutual wills after the death of the first individual.

Which of the following is (are) among the exceptions to the general rule that property given within three years of death is excluded from the donor's gross estate for federal estate tax purposes? (I) Life insurance (II) Gift taxes paid on the value of a gift (III) Farm land gifted to a son or daughter (IV) A gift of securities in trust with a retained life-income

(I), (II), and (IV) only Farm land gifted prior to death is excluded from the deceased's gross estate. Life insurance and gift taxes are both included in the gross estate when the gift takes place within three years of death. A gift of a retained life estate is also an exception.

Which of the following statements concerning powers of appointment are correct? (I) A power to appoint to oneself the annual contributions to a trust is a general power. (II) A power to appoint to oneself the lesser of $5,000 or 5% of the assets of a trust is a special power of appointment. (III) Failure to exercise a power to appoint up to 10% of the trust assets to the grantor's children will not result in a taxable gift. (IV) Failure to exercise a power to appoint to oneself 5% of the trust assets will not result in a taxable gift.

(I), (III) and (IV) only A power to appoint trust assets to oneself is a general power of appointment, even if the power is limited to only the annual contributions of additional assets. The power to appoint the annual additions to a trust is often called a "Crummey" power because of the decision in the Crummey case involving such a power. When the power is limited to $5,000 or 5% of the assets of the trust, it is still a general power. A power to appoint trust assets to the grantor's children is a special power, and the lapse of a special power is not a release of a power that is subject to gift tax. The failure to exercise the power, therefore, does not give rise to gift tax liability. The failure to exercise a general power of appointment is a lapse or release that is subject to gift tax. The Code, however, exempts a release or lapse of a power that is limited to $5,000 or 5% of the value of the trust assets. The failure to exercise this power does not result in gift tax consequences.

For which of the following reasons is the use of precatory language in a Will undesirable? (I) Such language consists of commands and directives. (II) Such language leaves the testator's intent unclear and subject to misunderstanding. (III) If stated as a wish, hope, or desire the will clause may not be enforced by the probate court.

(II) and (III) only Precatory language is a testator's expression of a wish, hope, or desire, rather than a command or directive. Precatory language is often disregarded by courts, because the intention is not clear.

Choose the most appropriate valuation technique to reduce federal estate taxes.

- A fractional interest discount can be taken for property the decedent held as tenants in common with a neighbor, who refuses to sell the property. A fractional interest discount can be taken at the property owner's death when the neighbor refuses to sell his partial interest in the tenancy in common property. A blockage discount only applies to sales of publicly traded stock that would depress market price if sold. A minority interest in a home is not a closely held business interest. A key man's discount is only available for the loss to the business upon a key employee's death.

Which of the following contract provisions effective at death will serve as will substitutes for the designated beneficiaries? (I) Employer based qualified retirement plan benefits for which the decedent named a child rather than the spouse as the designated beneficiary without spousal consent. (II) An IRA account for which the decedent named a child as the designated beneficiary. (III) Life insurance death benefits for which the insured named a child as the designated beneficiary. (IV) A provision in a pre-nuptial agreement naming a child to receive the decedent's estate and eliminating the spouse's right to any portion of the estate.

(II), (III) and (IV) only The benefits of a qualified retirement plan pass automatically to the surviving spouse, even though the spouse is not named as the beneficiary. The beneficiary designation for IRA accounts can provide for a child to receive the benefits instead of a spouse. Life insurance benefits can also be designated for children. A pre-nuptial agreement can also provide that a decedent's estate will pass to a child, and the spouse can relinquish an elective share.

In her will Mary left an apartment house to her husband, Fred, in trust with income payable to Fred for life. The terms of the trust provided that Fred could appoint the property in his will to any of their five children. Under these circumstances, which of the following statements is (are) correct? (I) Fred has a general power of appointment. (II) The value of the apartment will be included in Fred's gross estate. (III) The value of the apartment will not be includible in Fred's gross estate. (IV) If Fred could appoint only according to an ascertainable standard, there would be no change as to the property being includible in his gross estate.

(III) and (IV) only Fred was given a limited power of appointment, because he could appoint the property only to the five children and not to himself, his estate, his creditors, or his estate's creditors. The value of the apartment will not be included in his gross estate because Fred does not have a general power of appointment.

Which trust makes it mandatory for its corpus to be made available for distribution to the beneficiary when the beneficiary reaches the age of 21:

- 2503(c) trust The 2503(c) trust makes it mandatory for its corpus to be made available for distribution to the beneficiary at the age of 21. In the 2503(b) trust, the corpus need not be distributed to the beneficiary when the beneficiary reaches the age of majority. 2503(a) trust and 2502(c) trust are not applicable in this context.

To qualify for social security disability benefits on the wage earner's record, an ex-spouse must be?

- 50 or older - disabled and - married to the wage earner for 10 years or longer

Incomplete delivery situations involve transfers where certain technical details have been omitted or a stage in the process has been left uncompleted. Choose the incomplete transfers from the list given below:

- A check that is mailed in September 2012 and deposited in October 2012 - Gift of a negotiable note - A causa mortis gift when the donor is alive. - Joint bank account A personal check that is negotiated by a third party is a complete transfer. If a check is mailed in September 2012 and deposited in October 2012, The deposited check cannot be considered as a complete transfer. In order to be a complete transfer, the check must be cashed, so this is an incomplete transfer. If an individual makes a gift on his deathbed, which is called causa mortis, and is alive after that, then the transfer made is called an incomplete transfer. The creation of a joint bank account is an incomplete transfer because the donor has the power to withdraw money from the account and revoke the gift.

What is a durable power of attorney?

- A durable power of attorney empowers someone to act as your legal representative. - You can set up the durable power of attorney so that only a limited degree of legal power is transferred. - The durable power of attorney is separate from your will, and it goes into effect while you're alive but unable to act on your own

Who may be considered a skip person?

- A great-grandchild - A grand-nephew - A business partner who is 40 years younger than the transferor Skip persons can be grandchildren, great-grandchildren, grand-nieces, and grand-nephews. The skip person may or may not be related to the transferor. If a gift is made to a non-relative (not a wife) who is 37 ½ years younger than the transferor, like the business partner in this example, they are considered to be a skip person. A grandchild whose parent has died takes their parent's place in lineage and is not a skip person.

What is true regarding lack of marketability and minority interest?

- A lack of marketability discount can be combined with a minority interest discount when valuing limited partnership interests. - A lack of marketability discount is available for all interests, whether it is a minority or a majority interest. A lack of marketability discount can be combined with a minority interest discount for valuing limited partnership shares. A lack of marketability discount is available for all interests, whether it is a minority or a majority interest. A minority discount is available since minority ownership cannot influence the business, compel dividends, or liquidate the company. Both discounts apply to shares in closely held businesses, not to publicly traded shares on a stock exchange.

What is a disclaim property?

- A potential donee refuses to accept a gift

What is a grantor annuity trust?

- A split interest trust which provides the grantor with set amount of income based upon the initial value of the trust, for a set period of time.

What is a support trust?

- A support trust is a trust that contains a provision directing the trustee to pay to the beneficiary as much of the income and principal as is necessary for the beneficiary's education and support. The beneficiary's interest is unreachable by creditors to the extent necessary to support the "support" purpose of the trust. If a creditor claim would jeopardize the support function of the trust, then the attachment is void.

For generation-skipping transfer tax purposes, a direct skip includes:

- A transfer to a trust that has one great-grandchild beneficiary - A transfer to a trust that has three grandchildren beneficiaries A direct skip includes transfers made outright to a person who is two or more generations below the transferor, and transfers to trusts that include skip persons. A transfer to a grandchild whose parent has predeceased him moves up in lineage to take his parent's place, so a transfer to this child is not subject to GSTT. A trust that has both skip and non-skip beneficiaries is not a direct skip trust (it is an indirect skip trust.)

With respect to a transfer, which of the following is a skip person for generation-skipping transfer tax purposes?

- A trust benefiting grandchildren of the transferor A trust may be a skip person if all beneficiaries holding interests in the trust are skip persons, or no person holds an interest in the trust, but no distributions could be made to nonskip persons. If a trust is a skip person, its beneficiaries are not assigned to a generation.

What is pooled income funds?

- A trust generally created and maintained by a public charity rather than a private donor.

Which of the following is NOT an example of a generation-skipping transfer?

- A trust set up for spouse and children by a will The trust for spouse and children is the only example that did not benefit a skip person.

What is a generation-skipping trust?

- A trust that benefits several generations within the same family. It is established primarily to take advantage of estate and gift tax savings for the grantor.

What is a sprinkling or spray trust?

- A trust that permits the trustee to distribute income or corpus among various beneficiaries.

A trustee can be:

- A trustee can be an individual, either a professional or nonprofessional, or a corporate fiduciary.

Mr. Charles Edward died leaving his property to his son. His property consisted of two farms of equal value. Which of the following circumstances would prevent the executor of his estate from electing to use the special valuation rule?

- After debts and mortgages both farms equaled 40% of the gross estate One of the qualifications for applying the special valuation rule to farmland states that the value of the qualified property in the decedent's estate, less debts and unpaid mortgages, must equal at least 50% of the decedent's gross estate. The other factors do not affect the special valuation rule.

Robert Lester was a business tycoon who died wealthy. In his will he left all of his estate to his beloved wife Erica. Listed below are just four of the properties he owned outright at death. Which of them may be included in his gross estate?

- An apartment in Beverly Hills - Stocks of Sony Corporation - A gold Presidential Rolex watch - A private Cessna twin engine aircraft All types of property owned by a decedent outright at death are includable. This includes real and personal property, both tangible and intangible. Intangible personal property such as stocks, bonds, mortgages, notes and other amounts payable to the decedent are includable in the gross estate, as well as tangible personal property such as a decedent's jewelry and other personal effects.

What is a discretionary trust?

- An arrangement whereby property is set aside with directions that it be used for the benefit of another, the beneficiary, and which provides that the trustee (one appointed or required by law to administer the property) has the right to accumulate, rather than pay out to the beneficiary, the annual income generated by the property or a portion of the property itself

To read the case study for this question, click here. Mr. Chapman's choice of Darryl to be his attorney is ideal because he possesses the attribute that is the single most important distinction between a truly good estate planning attorney and an average practitioner. What is that attribute?

- An attitude of being counselor rather than advocate While a competent attorney must specialize in estate planning and administration, the single most important attribute required is compassion, an attitude of being counselor rather than advocate. The estate planning attorney must be keenly sensitive to the importance of the client's business, not only as figures on a ledger sheet but also as a personification of the client and as part of a goal-striving behavior. Such an attorney will be extremely conscious of the circumstances, needs, hopes and fears of the client and of his intended beneficiaries.

In 2012, how much exemption does the generation-skipping tax law permit per transferor for the value of any assets placed in a generation-skipping trust?

- An exemption of $5,120,000

Social Security Disability defines disability as:

- An inability to engage in any substantial gainful activity by reason of a medically determinable physical or mental impairment that has lasted or can be expected to last for a continuous period of 12 months, or result in death.

Every trust has an inclusion ratio that determines the portion of each future distribution or termination that will be subjected to the GST tax. Which of the following statements are correct?

- An inclusion ratio of "zero" means that the trust will always be exempt from the GST tax. - An inclusion ratio of "one" means that all taxable distributions and taxable terminations will be fully subject to the GST tax. An inclusion ratio of 1/2 means that one-half of all taxable distributions and taxable terminations will result in a GST Tax. GST tax does not pertain to a transferor's child, only to those who are two or more generations below the transferor. An inclusion ratio of "zero" means that the trust is totally exempt from the GST tax and an inclusion ratio of "one" means that all taxable distributions and taxable terminations will be fully subject to the GST tax.

Which of the following would trigger gift taxes?

- An incompetent's property transfer to a parent - A testamentary will contested by a beneficiary and a settlement exchange If a mother and daughter are in litigation and the daughter is claiming a large sum of money, a compromise payment by the mother to the daughter is not a gift. In a case where a widow settled with a son who threatened to break his father's will, the gift tax was levied. The gift tax can be applied even in the case of a transfer made pursuant to or approved by a court decree if there is no adversary proceeding. Gifts subject to tax are all those transactions where property or interests are gratuitously passed or conferred upon another, regardless of means or device employed. If the underlying cause was a divorce settlement or litigation, transfers of money do not amount to gifts.

To read the case study for this question, click here. If Bill Johnson's IRAs are left to Annette in 2012, which of the following are correct?

- Annette's gross estate will include the value of any IRA not consumed before her death. - Bill will include the FMV of the IRA account at death in his gross estate. - If Bill had a taxable estate, Annette can deduct a portion of the estate taxes attributable to the IRA on her income tax return). Bill's estate will be responsible for the estate tax since the IRA was an asset over which Bill held an element of control. Therefore, its full market value is included in Bill's estate for estate tax purposes. Annette will also be responsible for the taxes but any estate tax liability paid by Bill's estate on the income portion of the IRA is an income tax deduction to Annette as she withdraws the IRA income. Any amount of the IRA remaining at Annette's death will be included in her gross estate when she dies.

How would you define weakness in the context of estate planning?

- Any situation in the client's present estate plan that is in conflict with an estate planning objective

Transfers to a skip person can be protected from the generation-skipping transfer tax using all of the following except:

- Applicable credit The GST tax can be avoided by the annual exclusion, if it was exempt or excluded from gift taxes, and if the gift is completely sheltered by the GST exemption.

What is Sec. 2703?

- Applicable to any agreement or right to acquire or use the property at a price less than the fair market value

What is Sec. 2702?

- Applicable to interests in trusts

What is Sec. 2704?

- Applicable to lapsed voting and liquidation rights in a corporation or partnership.

What is Sec. 2701?

- Applicable to valuation of corporate and partnership interests designed for estate freeze purposes

What is the purpose of trying to qualify a stock redemption as a Sec. 303 redemption?

- Avoid being taxed as dividends IRC Sec. 303 allows a corporation to make a distribution in redemption of a portion of the stock of a decedent that will not be taxed as a dividend to use to pay death taxes and other expenses.

A standby trust is used when an owner is:

- Away from the country for four years - Temporarily disabled - Permanently disabled A standby trust takes effect when the owner is incapable of managing his assets. This may occur either because the owner has been rendered temporarily or permanently disabled. Such a situation may also arise if the grantor is overseas on business for a considerable period of time. A standby trust is not used when an owner is terminally ill.

In a valid trust it is a mandatory requirement that:

- Beneficiaries must be ascertainable in the trust agreement. - Beneficiaries have ascertainable identities if not named in the trust. In a trust it is essential for a beneficiary to be ascertainable. But it is not mandatory that the beneficiary be named individually. And there is no clause even remotely suggesting that the beneficiary ought to be a member of the family.

What is the benefit of ILIT?

- Benefits estate by reducing taxes and providing liquidity

Billy had set up a Q-TIP trust to take care of his vast property. He passed away in 2011. He provided a stream of income to his wife Clara from the trust corpus. In 2012, Clara too passed away. The property after Clara's death passed to Billy's son from a previous marriage. Which spouse determined who should receive the property in the Q-TIP trust when Clara died?

- Billy Billy (the decedent) used the Q-TIP trust because he wished to provide Clara (surviving spouse) with a stream of income payable from the trust corpus. And it also allowed him to have postmortem control over the property when Clara died. A Q-TIP can distribute income to the surviving spouse for her life yet allow the decedent to determine who receives the property when the surviving spouse dies.

Bad bargain is an exchange of property in an ordinary course of business. Bad bargain assumes that the transactions are:

- Bona fide - At arm's length - Profit- or loss-oriented

Mark places his assets into the corpus of a revocable trust. He will not escape estate tax liability or income tax liability. There are, however, some advantages and some disadvantages to placing property in a revocable trust. Choose the advantages associated with a revocable trust from the options given below:

- Capable of being flexible and terms of the trust can be altered, amended or revoked. - The grantor can revoke the trust entirely if he realizes that it does not fulfill his larger estate planning objectives. - In an event of the grantor's requirement, he can amend it so that it can be made available for him or his family. The primary disadvantage of a revocable trust is that the assets placed into the corpus of such a trust are included in the gross estate of the grantor and do not escape estate tax liability or income tax liability.

Which of the following may be circumstances where GST techniques can be applied?

- Client wishes to make a gift to grandchildren directly. - Client wishes to provide income for children and distribute assets to subsequent generations. - Client wants to avoid transferring assets to children's already large estates GST planning may help clients who have wealthy children and do not wish to increase their children's estates, clients who would like to create a trust to generate income for children but preserve principal for subsequent generations, and clients who would like to make a direct transfer to skip persons.

A trustee has the power to:

- Collect trust property, settle claims, sue or be sued. - Sell, acquire or manage trust property in a manner that is in the best interests of the trust. - Vote for corporate shares.

Beneficiaries are equitable owners of the trust in the sense that they are:

- Entitled to the enjoyment of the trust property - Entitled to the income produced by the trust property Beneficiaries can be considered to be equitable owners of the trust in the sense that they are to enjoy the property or its income. A trust arrangement is one in which one party (the trustee) holds legal title to property for the benefit of one or more beneficiaries. As a fiduciary, the trustee must hold the property, invest it, distribute its income, pay the trust's income tax liability, accumulate income, if necessary, and render any services required by law to ensure that the beneficiaries receive the enjoyment and use of the property in an equitable manner

Pick out the correct statements that distinguish between commercial annuities and private annuities. Click all that apply.

- Commercial annuities are sold by those in the business of selling annuities while private annuities are not - Commercial annuities use standard actuarial tables to determine payout terms, while private annuities use many other factors The person assuming the obligation for a private annuity is not in the business of selling annuities, in contrast to commercial annuities. To determine payout terms, commercial annuity companies use standard actuarial tables, whereas a private annuity may use many other factors. Commercial annuities are usually funded with cash, while private annuities are usually funded with various types of property, such as real estate or corporate stock. The value of the survivorship interest in a commercial annuity is the amount that the same insurance company would charge for a single annuity on the survivor at the time of the first annuitant's death. The value of a survivorship private annuity is determined using government valuation tables.

John Smith and his wife resided in Idaho for ten years. They acquired substantial property there. Thereafter they moved to New York. What is the status of the property they acquired in Idaho qualified as?

- Community property If the couple resided in a community property state at any time during the marriage and acquired property in that state, the property retains its status as community property even though the couple may have subsequently moved to a common law property state.

The tax and non-tax consequences of gifting property outright to others include all of the following:

- Complete transfer of ownership - Loss of income from income producing property - Transfer of basis - Loss of control over property interests Property interests cannot be disclaimed if the intended recipient actually takes possession the property.

Business continuation agreements take a number of forms. Which of the following are types of business continuation agreements?

- Cross-purchase agreement - Entity plan - Third party business buyout agreement The cross-purchase agreement is an agreement between the individual owners. The entity plan is an agreement between the business itself and the individual owners. The third party business buyout agreement is an agreement between the individual owners and a key person, family member, or outside individual. These are some forms that the business continuation agreement takes. Corporations commonly use stock redemption plans or shareholder cross-purchase plans.

Provide the steps for evaluating net interest?

- Determine the fair market value of all family-held interests in the entity - Subtract all family held senior equity interests, then subtract the value of all applicable retained interests held by the transferor and family members - Allocate the remaining value among the transferred interests and any other family-held junior or subordinate interests - Reduce value for consideration received, and for any minority or similar discounts

In a family limited partnership which of the following methods can be used by general partners to reduce transfer taxes when gifting limited partnership shares?

- Discounts for lack of marketability - Annual exclusions - Discounts for minority interests - Gift splitting with spousal consent - Use of general partners' applicable credit amounts

What are the characteristics of Terminable Interest Property?

- Donor cannot take a marital deduction for gift to spouse

What is the characteristic of 2034?

- Dower and curtesy interestA decedent's gross estate includes under section 2034 any interest in property of the decedent's surviving spouse existing at the time of the decedent's death as dower or curtesy, or any interest created by statute in lieu thereof (although such other interest may differ in character from dower or curtesy). Thus, the full value of property is included in the decedent's gross estate, without deduction of such an interest of the surviving husband or wife, and without regard to when the right to such an interest arose.

Which of the following documents must be drafted by an attorney?

- Durable power of attorneys - Grantor Retained Income Trusts - Charitable Remainder Trusts - Wills Attorneys must draft durable power of attorneys, wills and trusts. Couples in non-traditional relationships can write their own domestic partnership agreements, but each partner should have the agreement reviewed by their attorney.

What has an impact on the client's plans for distribution of his estate to family members, spouse or other beneficiaries?

- Estate tax liability - Gift tax liability - Income tax liability Substantial gift or estate tax liability, and a client's income tax liability have an impact on the client's plans for distribution of his estate. Property tax has no bearing on the client's plans for distribution of his estate.

Gifting assets to charity with significant appreciation potential allows for the reduction of which of the following?

- Estate tax liability - Income tax liability Lifetime gifts to a qualified charity of appreciating property remove all future appreciation of the property from the donor's gross estate. As a result, the donor has the ability to exercise some control over the amount of the estate tax liability. Additionally, lifetime gifts of assets, whether or not appreciating assets, may qualify for the charitable income tax deduction, which allows the donor to reduce his income tax liability.

The primary characteristic of which of the following trusts is the accumulation of income?

- Estate trust An estate trust is a trust established by the decedent which accumulates its income rather than distributing it to the spouse. The other trusts mentioned here provide the surviving spouse with a stream of income. They do not accumulate income.

Who is named in a valid will as the personal representative of the testator?

- Executor The executor is named in a valid will to serve as the personal representative of a testator when his or her will is being probated. A trustee and co-trustee are named in a trust agreement. An attorney is usually selected by the client for legal advice on estate planning but may also be appointed as executor, co-executor, trustee or co-trustee.

How is the estate treated with JTWROS with non-spouses?

- FMV of the property is included in a JT's estate adjusted for contribution

A CRAT is used in situations where the donor wishes to provide a noncharitable beneficiary with a stream of income to last for an unlimited number of years. State True or False.

- False A CRAT is used in situations where the donor wishes to provide a noncharitable beneficiary with a stream of income to last for a period of years, usually for the life of the income recipient or for a stated term of years. If a term of years is used, the period cannot exceed 20 years.

After full implementation, an estate plan can be said to be final. State True or False.

- False An estate plan is never final. Changes in the client's family situation, testamentary intentions, or the tax laws may make an estate plan that was at one time perfect an outmoded plan. It is for this reason that estate plans are monitored.

The greater the amount of property that is transferred inside the probate estate, the smaller the amount of administrative and legal expenses incurred by the estate. State True or False.

- False Costs involved in the probate of an estate are likely to be higher than if probate is avoided because most administrative and attorney's fees are based on a percentage of the value of the probate estate. Additional costs also may be involved if the probate procedure includes appointments of guardians, conservators, or appraisers. Therefore, the greater the amount of property that is transferred outside the probate estate, the smaller the amount of administrative and legal expenses incurred by the estate.

The term "irrevocable" is applied only to trusts that are established during the lifetime of the Grantor. State True or False.

- False Since all testamentary trusts are, by nature, irrevocable (since the Grantor has passed away when the trust takes effect and cannot be changed), the term "irrevocable" can be applied to trusts that are established at the Grantor's death or during the lifetime of the Grantor. Contrast that with a revocable trust which must be established only during the Grantor's lifetime. A revocable trust becomes irrevocable upon the Grantor's death.

Publishing a probate notice in a newspaper is enough to avoid litigation by unnotified creditors. State true or false.

- False The U.S. Supreme Court ruled that mere publication in a newspaper was and had failed to provide written notice or actual notice of the deadline for filing creditor's claims. Therefore, to avoid litigation by unnotified creditors, attorneys and other representatives of the estate should provide written notice or actual notice to all creditors of a decedent's estate.

To read the case study for this question, click here. Suppose that in 2012 Annette properly disclaimed the paintings by eminent artists worth $300,000. The law considers Annette has made no transfer of property for which tax purposes?

- Federal gift tax - Federal estate tax - Generation-skipping transfer tax It is considered that no transfer has been made by the disclaimant for federal estate, gift and generation-skipping transfer tax purposes even if local state law does not characterize the refusal as a disclaimer. The disclaimed paintings will be transferred to Annette's children, and will be subject to GSTT in Bill's estate.

The advantages of a revocable living trust include:

- Flexibility The ability of the grantor of a revocable living trust to revoke the trust leads to the inclusion of the corpus and all undistributed income in the gross estate of the grantor. As a result it is high on flexibility but is not effective as a tax saver.

The advantages of a revocable trust include:

- Flexibility - Easily amendable The main advantage of a revocable trust is that it provides the grantor with flexibility if the corpus needs to be consumed by the grantor or the grantor's family. It also provides the grantor with greater ease in amending the trust if the grantor is unhappy with the way in which the trust is operating. The primary disadvantage of a revocable trust is that the assets placed into the corpus of such a trust are included in the gross estate of the grantor and incur both estate tax and income tax liability.

What is the generation assignment for same generation?

- Friend who is 10 years younger - Spouse

To determine the adjusted gross estate, which of the following may be subtracted from the gross estate?

- Funeral expenses - Administration expenses - Estate maintenance expenses - Debts and taxes owed by decedent

Choose all options that are true regarding GST tax.

- GST tax is in addition to any lifetime taxable gifts made to skip persons. - But under the deceased parent rule (IRC § 2651(e)), descendants are moved up to their parent's level if the parent dies before the date of transfer. Thus a grandchild who might be a skip person to his or her grandparent will not be treated as a skip person if his or her parent dies before the grandparent - A GST tax must be paid from assets that are subject to the federal estate tax if a direct skip is made at death. - The total tax will be greater than 100% if the applicable rates of the gift or estate tax are 55%.

A disclaimer that results in a transfer to or for an individual who is two or more generations younger than the transferor can attract:

- Generation-skipping transfer tax A transfer to an individual who is two or more generations younger than the transferor is a direct generation skip transfer. A disclaimer can create generation-skipping transfer by someone other than the disclaimant because, under GST tax law, a direct skip transfer is a GST subject to the GST tax if it is also subject to federal estate or gift tax.

The Pastinos own a popular bakery in their neighborhood. Their daughter, Maria, is a divorced mother of two children, struggling to make ends meet. The Pastinos would like to remove their building and land from their estate, along with the future appreciation. They would also like to give Maria $20,000 a year in additional support but they are "cash poor" since their wealth is tied into their business. What is the best transfer technique to use?

- Gift Leaseback A gift leaseback is the best transfer technique to use since a gift of the bakery building and land would remove these assets and appreciation from the Pastinos' estate. Lease payments to Maria would give her $20,000 a year in additional support, and are deducted by the bakery. A business continuation agreement guarantees the sale of a decedent's business interest to partners or a corporation. A GRAT is a split interest trust that provides income to a grantor for a period of time. A private annuity would provide income to the Pastinos for life, instead of to their daughter.

Robert resides in Texas but has some real estate in Florida. He wants to avoid the ancillary probate procedure while disposing of his real estate in Florida. What options does he have?

- Gift the real estate to another person. - Place the real estate in a trust at some point in his lifetime. Robert has only two options — either to gift the real estate to another person, or to place it in a trust at some point in his lifetime to avoid ancillary probate procedure. He cannot choose the other two options because for a will to dispose of real estate located in a state other than that of Robert's residence, the property has to be subjected to ancillary probate procedure. In his will, Robert can direct who should get the real estate in Florida, but an ancillary probate procedure will be necessary. A provision in the will that attempts to dispose of the property in an attempt to avoid an ancillary probate procedure is null and of no effect.

What is the characteristic of 2035?

- Gifts made within three years of death

Financial competence of a beneficiary includes the ability of the beneficiary to:

- Invest in and manage assets - Provide for other family members - Supervise and administer assets Financial competence of a beneficiary is reflected by the beneficiary's ability to invest in and manage assets, provide for family members and supervise and administer assets. Earning $100,000 per year is not a criterion.

Which of the following distributions are not subject to the throwback rules:

- Gifts or bequests of a specific sum of money or property - Distributions that do not exceed the trust's accounting income. Gifts or bequests of a specific sum of money or property and distributions that do not exceed the trust's accounting income are not subject to throwback rules. Distribution of income that accumulates before the beneficiary's birth and before the beneficiary reaches the age of 21 are not subject to throwback rules. A specific amount set aside, paid or used for specific charity alone qualifies for non-imposition of throwback rules.

What is the characteristic of 2037?

- Gifts that were transferable only after the death of donor

What is the characteristic of 2036

- Gifts where donor retained the life estate

What is the generation assignment for intermediate generation?

- Grandchild whose parent died before - Children - Friend who is 20 years younger

In a generation-skipping trust, who gets the outright title to the property upon the death of the grantor's children?

- Grantor's grandchildren In a generation-skipping trust, on the death of all of the children, the outright title to the property in the corpus passes to the grandchildren of the grantor. It does not pass on to the trustee, grantor's siblings or to a charity.

What is the generation assignment for skip generation?

- Great niece

Harold Bates would like to gift some part of his stock in the corporation to an applicable family member, after which he would retain some interest in the corporation. To which of the following members could he gift a part of his stock?

- Harold's wife Sandra - Harold's son Jake Harold can gift the stock to Sandra and Jake. Under Sec. 2701, family members are the transferor's spouse, lineal descendant of the transferor or spouse, and the spouse of any descendant.

Matthew Perry transfers stock in Perfect Engineering Company to a trust, with the provision that income from the trust is payable to himself for life. He also specifies that the corpus of the trust would pass on to his daughter after his death. The Perfect Engineering Company stock would be includable in Matthew's estate because:

- He has retained a life estate Under Section 2036 value of property transferred as a gift is includable in the donor's gross estate if the donor retained life estate. The underlying principle for including this type of lifetime transfer is that the right to enjoy the income of a property is characteristic of ownership. The donor will continue to be treated as owning the property because the donee's possession or enjoyment of the property cannot begin until the decedent dies and transfers the retained interest.

To read the case study for this question, click here. We can say that Caleb is competent to serve in his role as executor of Mr. Chapman's estate if:

- He is legally capable to be an executor. - He has the intellectual ability required of an executor. - He has the emotional capacity required of an executor. Competence encompasses the legal ability to serve as well as the intellectual and emotional capacity to serve effectively. Funds are not required to be an executor.

What are the possible tax implications for a daugther who received an inheritance from her father and disclaimed it, so that it then passed on to a charity?

- Her father's estate may receive a charitable deduction.

When Sharon was 68, she was diagnosed as having terminal cancer. Doctors gave her a life expectancy of just eight months, with little hope of a cure. She had four children to whom she gifted the property that she owned outright. She died a year later. Which of her properties that she gifted as follows would be includable in the gross estate?

- Her villa to Stephanie, retaining life estate - Her country house to Martha, for life, at Martha's death, it returns to Sharon, unless Sharon is dead, then remainder to Martha's issue - Her shares in Oracle Corp., to Winston, retaining power to revoke As per IRC Section 2035, gifts made within three years of death are not includable. The exceptions are properties includable under IRC Sections 2036, 2037, 2038 or 2042. The villa falls under Section 2036 because Sharon retains life estate. The country house as life estate falls under Section 2037 because it is contingent that Martha be alive to enjoy the gift, with Sharon still retaining a right to regain the property personally. Her shares in Oracle Corp., fall under Section 2038 because Sharon retained the power to revoke. Therefore, these three properties-the villa, country house and Oracle shares-are includable in the gross estate. The only property that is not includable is the garment factory, which was a complete gift.

Dennis Vaughan died on January 18, 2012. His estate consisted of several properties, all of which he had willed to his wife and three children. The executor elected alternate valuation for federal estate tax purposes, which would be the value as of July 18, 2012. Which of the following properties were valued correctly?

- His car was valued at the market price of $20,000 on July 18, 2012. - His business that his eldest son now runs was valued at $500,000 on July 18, 2012. Once the alternate valuation is selected for valuation purposes, such date applies to all assets in the estate. However, only the car and his business were valued correctly as on July 18, 2012. The alternate valuation provision specifies that if the property is sold within six months of the decedent's death, it will be valued as of that date, not the six-month date. Based on this, the correct valuation of the country house would be $80,000, for which it was sold, and not its market value of $65,000 as on July 18, 2012.

A family limited partnership must have a legitimate business purpose to exist. The types of property that can be transferred into a FLP include which of the following assets:

- Investment real estate - Non-voting stock of a closely held business Investment real estate and non-voting shares of a closely held business can be transferred into the FLP. The FLP cannot hold S Corporation stock, and can only hold securities that are less than 80% of the value of the FLP property.

Arnold Fox picked up a booklet at his attorney's office that had a section on computing federal estate tax. Some of the steps in the computation were in the wrong order. Select the correct order. I. gross estate value II. adjusted gross estate III. estate tax payable before credits IV. taxable estate V. net federal estate tax.

- I, II, IV, III, V. The adjusted gross estate must be calculated before determining the taxable estate. The correct sequence for computation of federal estate tax is (1) gross estate value (2) adjusted gross estate (3) taxable estate (4) estate tax payable before credits (5) net federal estate tax.

What are IRD assets?

- IRA accounts - 401(k) plan accounts - Salary - Deferred annuity - Sales commission

With respect to disclaimers, which of the following is true?

- If property is disclaimed by a person in favor of a surviving spouse or charity, the marital or charitable deduction will be permitted provided the property would otherwise qualify for these deductions. Disclaimed property acts as though it never went to the disclaimant's estate. If the property qualifies, it can take advantage of marital or charitable deductions. A surviving spouse is also eligible to make a disclaimer. However, a disclaimer is not treated as a gift for gift tax purposes. The property disclaimed will not be included in the disclaimant's estate at death as a transfer with retained life estate. Income received on the disclaimed property will be chargeable to the person in whose favor the property was disclaimed and not the disclaimant.

When is the client's estate subject to ancillary probate?

- If the client owns real property in more than one state - If the client has no lifetime plans for the disposition of the real property owned in other states - If the client has no lifetime plans for transfer of the real property owned in other states If the client has property in more than one state and has no lifetime plans for disposition or transfer of the property, the client's estate will be subject to ancillary probate for any real estate located in a state other than the state of the client's domicile.

For federal estate tax purposes, an unmatured life insurance policy owned by the decedent on the life of another is valued as follows:

- If the policy is paid up, the policy is valued at its replacement cost The value of an unmatured policy owned by a decedent on the life of another is includable in the gross estate where he or she predeceases the insured. If the policy is paid up, its value is its replacement cost. If the policy is new, the value is the gross premium paid. If the policy is a term policy, the value is the unused premium.

The differences in state law that can influence the creation of an estate plan are the following:

- If the state has a mortmain statute - If the state permits holographic wills - If the state has state inheritance taxes State laws in the different states affect estate planning. Factors such as whether the state has a mortmain statute, whether the state permits holographic wills and whether the state has state inheritance taxes, have an effect on estate planning. A monographic will is not applicable, as it is not a valid term in this case. As there are fifty different states in the U.S., an individual should, while planning his or her estate, consult an estate planner.

When is a special hearing of a trust conducted?

- If the trustees do not agree upon a unanimous course of action for the trust If the trustees cannot agree upon a consensus course of action for the trust, a special hearing is held in order to determine the effectiveness of the trust. Special hearings are not held when trusts are terminated or formed. The death of a beneficiary does not call for special hearings, in this context

What are the characteristics of gift tax?

- Imposed only on the transfer of property or an interest in property

Which of the following are requirements for a disclaimer?

- In writing - No later than 9 months after transfer creating interest Disclaimers must be in writing, irrevocable, declared no later than 9 months after the transfer, and the disclaimant cannot have a say in who receives the property.

How is the estate treated with JTWROS with spouses and non-spouses?

- Income is split equally among all tenants owning property held as JTWROS

Carol O'Neil, age 61, has a profitable graphic art business valued at $800,000, and personal property worth $250,000. Carol wants to retire from her business, and needs $3,000 per month in retirement income. Her niece, a recent art school graduate, would like to buy Carol's business, but she doesn't have enough money for a down payment. What is the best transfer technique to use?

- Installment Sale An installment sale is the best transfer technique to use since Carol can sell the business to her niece with no down payment, and receive secured income payments to meet her retirement needs. The PV of any outstanding installment payments will be included in her estate at death, but this will not trigger an estate tax liability. A private annuity is based on unsecured payments, and Carol needs to receive the income or take back the business if needed. A sale leaseback is a device used to provide cash to buyers, and an IDGT is a sale of the business to a trust to permanently remove the property from a grantor's estate, which is not an important tax consideration for Carol.

Julia Douglas at 55 would like to hand over the reigns of her multi-billion dollar enterprise to her son Eddie. But as he is very new to the business, she would like to give him some more time to learn the tricks of the trade. She plans to freeze growth of the enterprise and pass on the benefits to Eddie. Which of the following methods can she adopt to bring about the freeze?

- Installment sales of business interests to family members - Use of the private annuity - Preferred stock recapitalization - Buy-sell agreements There are a variety of freeze growth techniques that Julia can use. Installment sales, private annuities and preferred stock recapitalizations give business owners cash flow and they transfer the growth in the value of the enterprise to the next generation. Buy-sell agreements also freeze the value of the business interest for estate tax purposes.

What is partial disability?

- Is defined in occupational terms with reference to time and duties.

Pick out the correct statement(s) regarding codicils from the alternatives given below:

- It is a more convenient method of revising a will, since it can be used to make changes in a will. - Codicils are relatively simple, as most codicils are only one or two pages in length. - Making a codicil is not as costly as revising the entire will.

Which of the following statements accurately describes a disclaimer trust?

- It is inserted in the will of the decedent. - It will function only if the surviving spouse disclaims the bequest. A disclaimer trust is inserted in the will of the decedent, and it will function only if the surviving spouse disclaims the bequest. The trust may be for the benefit of the spouse and the children, or only for the children. Its purpose is to allow the surviving spouse to take advantage of all or part of the decedent's estate tax exclusion amount, the applicable credit.

An Intentionally Defective Grantor Trust (IDGT) is considered "defective" due to which of the following circumstances:

- It is not a separate income tax entity from the grantor. The IDGT is considered "defective" because it is not a separate income tax entity from the grantor. Grandchildren can be trust beneficiaries without GST taxes imposed on the sale of property to the trust. The property is sold to the trust at its fair market value, which is tax-neutral, and is not a recognized taxable gain or loss for the grantor.

Why is a will still the most important estate planning document?

- It is only within the provisions of the will that the decedent's can appoint the executor of the estate - If the decedent's has minor children, it is only within the provisions of the will that the guardians for the minor children may be appointed

What is an estate trust?

- It provides income to the grantor's spouse only when the trustee deems it proper.

Identify the features applicable to a testamentary trust: 1. It takes effect upon the death of a grantor. 2. The trustee in such a trust can use discretionary powers to use money in appropriate fashion. 3. It is always in writing. 4. Such trusts can bypass probate.

- It takes effect upon the death of a grantor. - The trustee in such a trust can use discretionary powers to use money in appropriate fashion. - It is always in writing. Testamentary trusts can be created as part of a decedent's last will and testament, or within the provisions of a revocable living trust. They are always in writing. Since they endorse powers by a testament, they have to go through the probate process.

To read the case study for this question, click here. Bill Johnson's executor will have to file Bill's final income tax return. His taxable year is:

- January 1, 2012 - October 21, 2012 The tax year starts on January 1, 2012. A decedent-taxpayer's tax year ends with the date of his death, which for Bill Johnson is October 21, 2012.

To read the case study for this question, click here. Mr. Chapman has an expensive collection of porcelain figurines. Whenever members of his family traveled they brought back gifts to add to his collection. Caleb knew the value of his uncle's collection because he had helped him research about the history of various figurines. Which of the following attributes required of a good executor did Caleb display in this regard?

- Knowing the nature, value and extent of the assets An executor must be familiar with the nature, value and extent of the assets in the estate. Many individuals invest in or amass collectibles for pleasure. An executor with expertise in this area would know how to safeguard, transport, insure, have appraised and sell such items far more readily than an executor with no particular knowledge of the subject matter.

Some Activities of Daily Living (ADLs) found in LTC policies include:

- LTC insurance policies are not standardized, but most contain ADLs such as eating, bathing, dressing, walking, toileting, continence, transferring and taking medicine.

Which of the following are concerns for selection of a trustee?

- Last for multiple generations - Possible law suits - Tax sensitive - Conflict of interest Trusts can last for multiple generations; therefore, it is necessary to pick successor trustees. Financial security and checks and balances are necessary in case of law suits. Revocability of trusts dictates whether or not the trustee is liable for the taxes of the trust. The trustee's relationship to the beneficiary may raise conflict of interest in his or her actions.

Why is it important for unmarried couples that own property together as tenants in common to obtain separate wills?

- Laws of intestacy only pass property to related heirs, therefore each partner must obtain a Will to bequeath separate property interests to one another.

Disadvantages of a power of attorney are:

- Lending institutions may not recognize the authority of an agent under a durable power of attorney - Durable powers of attorney cannot be used after death to dispose of property omitted from a will Durable powers of attorney have limitations, such as having third parties recognize an agent's authority, and that the power ends at the principal's death. It is a safeguard for the principal that the agent cannot exceed the powers contained in the power of attorney document, and is an advantage that the power cannot be revoked if the principal becomes incapacitated.

A mortality table is used when valuing:

- Life estate - Reversion - Annuities The same method of valuation is used for an annuity, a life estate, a reversion or a remainder. A mortality table is used to convert one of the unknowns to a known value. Here the length of the lifetime involved is converted to life expectancy. The mortality table is based on a study of the longevities of a large number of people over a selected period of time, which indicates the expectancy of life in years for a mythical individual who represents the average experience at each age of life. Life insurance and real and personal property do not use mortality tables for valuation.

What provides annual exclusions?

- Limited partnerships - 529 plan - crummey power

Nicolas is thinking of gifting his old family home to a qualified charity. The family home has actually depreciated in value. Its present FMV is less than what Nicolas had paid for the home. As his financial planner, Nicolas is asking you to outline planning strategies, including gifting, which should be considered. Given these limited facts, which strategies would not apply?

- Make a completed gift of the property to the charity. If managing taxes is a concern, it may be better for Nicolas to keep the property. As his old family home (the property being considered for gifting) has depreciated in value so that its present FMV is less than his basis in the property, he might be wise to keep the property.

What are the major types of trusts used in estate planning for married couples?

- Marital trust - Power of appointment - QTIP/ QPRT - Non-marital trust - Estate trust Remember, marital trust includes the power of appointment, QTIP, QPRT and estate trust.

What are the possible tax implications for a son who received an inheritance from his father and disclaimed it, so that it then passed on to his mother, the decedent's spouse?

- May qualified his dad's estate for a marital deduction due to the transfer of the assets to the surviving spouse.

Of the forms discussed in this module, which form lists insurance agents, investment advisors, personal physician and business associates and their contact information?

- Miscellaneous Information Form The miscellaneous information form includes the basic information about the spouse's occupation, employer's and social security numbers. The form also lists insurance agents, investment advisors, personal physician and business associates and their contact information.

Mr. Gerald Holmes transferred his estate to a trust created for his daughter Claire and appointed a trustee, Mr. Remington. He specified that the income may be accumulated or distributed to Claire until she reaches age 25, at the discretion of Mr. Remington, according to what he determines to be in the best interests of Claire. She would receive the entire principal and any accumulated income at age 25. Mr. Holmes retained power to remove Mr. Remington as trustee and appoint another, other than himself. Mr. Holmes died when Claire was 22. Which of the following is true?

- Mr. Holmes does not retain the power to revoke but does retain the power to remove the trustee at will. Therefore, the value of the trust is included in Mr. Holmes estate According to IRC Section 2038, if the decedent can alter, amend, revoke or terminate property transferred to others, change beneficiaries, hasten the time beneficiaries can receive property, change the amount of property allocated to beneficiaries, appoint himself trustee, appoint a successor trustee if the trustee resigns, or remove a trustee at will, the property subject to the power is included in the decedent's estate.

What type of trust would a client and spouse with managerial experience but lack the desire use?

- Name a professional trust company to manage the assets with a family member named as co trustee.

What type of trust would a client and spouse without managerial experience use?

- Name a trust company or other corporate trustee as the party responsible for managing assets and making investment decisions.

Divorced parents want assurances that former spouses will not have access to property, trusts or custodial accounts if that parent dies. Which methods will accomplish this goal?

- Naming someone other than the ex-spouse as custodian of a UGMA or UTMA account - Creating irrevocable trusts that delay the distribution of corpus to children until after the death of a former spouse - Distributing trust property to contingent remaindermen such as a current spouse or other children, if the minor beneficiary dies.

Within how many months of the decedent's death is the federal estate tax payable?

- Nine months The federal estate tax is imposed on the decedent's estate and is usually payable by the decedent's executor on the date the return is due, that is, within nine months of the decedent's death.

Doug and Lara want to gift property during their lifetime to reduce their taxable estates. They also want to avoid gift tax liability by not using any of their lifetime gift tax exclusion. They consult their financial advisor, who recommends making a split gift. Do you think the financial advisor made the right recommendation?

- No Doug and Lara may elect to split a gift in order to reduce their gift tax liability. However, the split gift allows each of the spouses to use their own annual exclusions and lifetime gift tax exemptions. If these clients are interested in making lifetime gifts without using the lifetime gift tax exclusion, then Doug and Lara should consider making lifetime gifts to a qualified charity. As a result of the unlimited charitable gift tax deduction, an unlimited amount of assets may be gifted to a charity without the payment of any gift tax liability.

If George makes a gift of an original Da Vinci painting to the Boy Scouts of America, but specifies that he is allowed to keep the painting if the charity fails to use it for the Army Welfare Fund, will the gift qualify for a gift tax charitable deduction?

- No George did not make an irrevocable transfer of property to the Boy Scouts of America (qualified charity). In order for the transfer to be considered effective, George must irrevocably part with all ownership or control of the property. If there are "strings attached" or conditions specified by the donor under which the property may be used or enjoyed by the donee, then a completed gift has not occurred. Therefore, the gift will not qualify for a charitable gift tax deduction, nor will it qualify for an income tax deduction in the form of the charitable contribution deduction.

Tom purchased an annuity contract for his son, with payments to be commenced at his death, which was irrevocable. When Tom dies, which of the following apply to the value of the annuity interest? Click all that apply.

- Not includable in Tom's gross estate because he has no beneficial interest in the payments. - Not includable in Tom's gross estate because he has no right to payments. Both IRC Sections 2039 and 2033 are applicable here. Tom does not have any beneficial interest in the payments because they will commence at his death and go to his son. The annuity is not being transferred at death. Therefore, it is not includable in Tom's gross estate. According to IRC Section 2039, it will be includable only if it provided the decedent with a payment or a right to payment for life.

Consequences of using the marital deduction to completely offset estate taxes include all of the following:

- Over-qualifies the estate for the marital deduction - Prevents the decedent's estate from using the alternate valuation date - Includes the decedent's property and all future appreciation in the unmarried surviving spouse's estate at death - Prevents the owner from selecting beneficiaries to receive the property after the surviving spouse's death Property passing to the surviving spouse through the marital deduction is not protected from creditor claims.

Disability contracts typically define disability in three ways. Choose the best definition to have in a disability policy.

- Own occupation definition An own occupation clause deems the insured to be totally disabled when they cannot perform the duties of their regular occupations. The insured can be at work in some other capacity and still be entitled to benefits. Any gainful occupation definition means that insureds are totally disabled when they cannot perform the duties of any gainful occupation for which they are reasonably suited because of education, training or experience.

What are the proceeds from a Sec. 303 stock redemption used for?

- Pay estate taxes as well as administrative and funeral expenses In a Sec. 303 stock redemption, the owner's stock passes to his estate. Next, the corporation receives the proceeds on his life insurance. It uses these proceeds to pay the owner's estate for qualifying stock for redemption. The estate transfers amount worth the insurance amount to the corporation. With this cash the estate pays the federal and state death taxes, and administrative and funeral expenses.

Which form details the information about pre or post-nuptial agreement, living will, and gift under the Uniform Gift to Minors Act?

- Planning Form The planning form covers general questions, such as, do you have a pre or post-nuptial agreement. Have either client created a living will? Have the clients made a gift under the Uniform Gift to Minors Act?

The powers of a trustee may vary from state to state. However, the general powers that a trustee may hold include the following:

- Power to sell, acquire or manage trust property in a manner that is in the best interests of the trust - Power to borrow money and use the trust corpus as collateral, if approved by the court - Power to receive additional assets into the corpus of the trust

Max Stevens, age 59 owns 100% of the common stock in a closely held software company valued at $3 million. His total estate including his company is worth $4.9 million. His company is growing rapidly, and he wants to share some ownership and future appreciation with his three children who work in the business. His other objectives are to receive income, reduce his estate tax and keep management control of the business. What is the best transfer technique for Max to use?

- Preferred Stock Recapitalization If Max recapitalized his stock into non-voting common and voting cumulative preferred shares, he could gift non-voting common shares to his children to reduce his estate and give them some ownership in the business. By keeping the voting shares, Max would maintain control of the business and fixed dividends from the preferred shares would provide him with income and a "qualified payment" under Chapter 14. A bargain sale of business property and an intra-family loan are completed transfers that would remove property from Max's estate, but he would not maintain control over the transferred property.

Ted Allard, age 72, owns 1/6 interest in a ski resort in Vermont, which is currently valued at $1.5 million. The ski resort continues to appreciate every year, and Ted's estate is now worth $4.5 million. Ted wants to give his ski resort interest to his son Bob, a wealthy portfolio manager, to remove it from his estate without paying gift taxes. However, he needs the income it generates for the rest of his life. What is the best transfer technique to use?

- Private Annuity A private annuity is the best transfer technique since the entire $1.5 million interest in the ski resort plus the appreciation will be removed from Ted's estate. Payments are unsecured, but Bob is wealthy and can presumably make payments throughout Ted's life. A preferred stock recapitalization is a gift of partial business interests that would be subject to gift tax. A bargain sale results in a gift tax for the difference between the sale price and the fair market value. An installment sale would cause the PV of any outstanding installment payments to be included in Ted's estate at death.

To read the case study for this question, click here. Upon his uncle's death, what are the responsibilities that Caleb will have to fulfill?

- Probate Mr. Chapman's will - Collect Mr. Chapman's property - Pay debts, taxes and expenses of Mr. Chapman's estate The main responsibilities for Caleb as the executor of his uncle's estate are to probate Mr. Chapman's will, collect his property, pay off the debts, taxes and expenses on the estate, and distribute the remaining assets according to his will. A trustee is responsible for reducing or eliminating probate costs and taxes. An attorney's duty includes organizing the entire estate planning team and gathering data about the client's financial goals.

The factors for selection of an estate planning technique that achieves the client's objectives include:

- Profession of the client - Marital status of the client - Health and life expectancy of the client The client's marital status, health and life expectancy affects the selection of an estate planning technique that achieves the client's objectives. Educational qualifications have no bearing on the selection of an estate planning technique-although, if a client is educated, it is easier to explain the strategies and techniques being considered. Also, from a creditor protection standpoint, one aspect of estate planning, the profession of the client is VERY important in selecting the appropriate estate planning technique or strategy.

What is incomplete gifts in trust?

- Properties transferred to revocable trusts

Which of the following properties are not includable in gross estate?

- Property gifted within three years of death of decedent - Property to which decedent holds the legal title but has no beneficial interest Property gifted within three years of the death of donor is generally not included in gross estate. The exceptions are found in IRC Section 2035 and pertain to relinquishing life estates, reversionary interests, revocable trusts and life insurance policies owned by the insured.

What is cancellation of notes?

- Provides a simple means of giving gifts to a number of donees of property that is not readily divisible.

Liquidity planning pertains to which of the following:

- Providing estates with sufficient funds to pay for taxes and expenses Liquidity planning is used to provide estates with sufficient funds to pay for estate administration expenses, debts, taxes and probate costs.

Suppose Ralph wants to provide his wife Clara with a stream of income (after his death) that terminates upon her death. Ralph also wants to determine who the ultimate beneficiaries of the property will be upon the death of his wife. Which type of trust will be appropriate in order for Ralph to fulfill his wishes?

- Q-TIP trust - Nonmarital trust A nonmarital trust or Q-Tip trust is appropriate when the decedent (in this case, Ralph) wants to provide the surviving spouse (Clara) with a stream of income that terminates upon the survivor's death and if the decedent wants to determine who the ultimate beneficiaries of the property will be upon the death of the surviving spouse.

To read the case study for this question, click here. Which of the following are the duties of the Northern Fiduciary Trust Company as the trustee?

- Reduce or eliminate taxes - Reduce or eliminate probate costs The goal of a trustee is to reduce or eliminate income or estate or generation-skipping transfer taxes at the federal or state level, as well as probate costs. Probating the decedent's will and distributing his or her assets is a duty of the executor.

As an alternative to the freeze, Benjamin retains common stock and transfers the preferred interest. Under which of the following processes can this be done?

- Reverse freeze The reverse freeze is a technique wherein the transferor retains common stock or partnership interests and transfers the preferred interests. Sec. 2701 is not applicable in this case. When Benjamin dies, the value of the common stock reflects growth, but is substantially discounted in value because of the burden of the preferred stock with its prior claim to dividends and liquidation proceeds.

For which type of trusts does it not matter who is selected as trustee?

- Revocable trust A revocable trust has a neutral impact on taxes at both federal and state levels. If the grantor has reserved the right to revoke the trust at any time, in most situations he or she will be treated as if he or she had not established a trust and had remained as the outright owner of the property in the trust. So it does not matter for tax purposes who is selected as trustee. But if one of the major reasons for establishing an irrevocable trust is to save taxes, the identity of the trustee is a very tax-sensitive decision, because inclusion of the trust's assets in the income, gross estate or generation-skipping tax base will result if the wrong party is selected as trustee.

If a $1,000,000 marital deduction was allowed at Sam's death for property left to his wife, Sadie, at Sadie's death the value of such property at the time of her death will be included in whose estate?

- Sadie At Sadie's death the value of such property at the time of her death will be included in her estate.

Nicholas Berg gave Samantha his beach house for life and remainder to her husband Dave. Dave dies while Samantha is alive. Which of the following statements is not true?

- Samantha is still alive, so Dave's remainder interest terminates at his death Though Samantha has interest in the beach house as life estate, when Dave dies his gross estate will include the value of his remainder interest, because it does not terminate at his death. An interest will be includable even if it's limited, contingent on something else or extremely remote, as long as it is does not end when the decedent dies. The contingency or remoteness of the interest will affect its valuation.

The late Mr. Daniel Sherwood bequeathed his property to his niece as "To Sandra Sherwood for life, remainder to her issue and if no issue then to the American Red Cross." Sandra died childless in 2012. At the time of Sandra's death:

- Sandra's life estate terminated. - Sandra did not own the property. Under IRC Section 2033, no inclusion is required for property in which the decedent's interest was obtained from someone else and was limited to lifetime enjoyment. Estate tax is a tax on the transfer of property at death. This means that if an interest terminated at the decedent's death and the decedent had no right to transfer at death, its value will not be included. Sandra did not have anything that she could transfer. Her life estate, by its terms, terminated with her death so there was nothing to transfer. Though Sandra had an interest in the property at the time of her death, it was not a property interest that she could pass to someone else.

To read the case study for this question, click here. Annette chooses to exercise her right to disclaim the bequest of the mushroom farm in favor of her only child, Wilson. She is treated for federal tax purposes as though:

- She predeceased her father. - She never received the property. A disclaimant is treated for federal tax purposes as never having received the property. It's as if he or she predeceased the transferor. Therefore, no transfer is considered to have been made by the disclaimant.

Arnold wants to leave his entire estate, worth $20 million, to his nephew Kevin. But Arnold is afraid that Kevin may not spend it wisely. Which type of trust should Arnold ideally arrange to alleviate his concerns?

- Spendthrift trust Provision of income to a specified beneficiary (in this case, Kevin) who cannot control money, or is improvident and cannot manage money wisely, is possible by structuring the trust in such a way that the beneficiary (Kevin) cannot transfer his or her interest in the income until it is actually received. A spendthrift trust does exactly this. In this way, the beneficiary's creditor cannot reach the trust or the income produced by the trust until the income is actually distributed to the trust beneficiary.

Statement A: Gibson makes a gift of a collection of original works of Manet to the National Museum. Statement B: Gibson makes a gift of a collection of original works of Manet to the local orphanage? Select the correct alternative(s) from the following:

- Statement A is a use-related donation of tangible personal property while Statement B is a use-unrelated donation of tangible personal property. The distinction between use-related and use-unrelated tangible personal property depends on the purpose of the charitable organization. As the property donated (original works of Manet) to the National Museum could be used directly by the charity itself, the donation is use-related. The same original works of Manet, if donated to an orphanage, are not related to the orphanage's purpose. Since the purpose of the orphanage is not to exhibit the works of art, it will sell the art and use the proceeds from the sale for its own purposes. As a result, this gift to the orphanage would be classified as use-unrelated.

Which of the following are types of business continuation agreements?

- Stock redemption agreements - Cross-purchase agreements The two types of business continuation are stock redemption and cross-purchase agreement. Stock redemption agreements are those in which the business itself purchases the business owner's interest. Cross-purchase agreements on the other hand are those in which the new or other existing owners acquire the shares.

Which of the following would most likely be allocated GST tax exemption?

- Stocks GST is likely to be allocated to assets that will appreciate in value such as stocks.

What are NonIRD assets?

- Stocks - Mutual funds - Bank CDs - Bonds

What is direct skip?

- Tax exclusive, transfer from individual to someone who is two or more generations younger

What is taxable distribution?

- Tax inclusive, income or principal is distributed to a skip person

What is taxable termination?

- Tax inclusive, transfer to skip person triggered by death of non-skip person

The Throwback Rule is designed to:

- Tax the beneficiary of a trust that accumulates income. - Tax that portion of the accumulated income as if the income has been distributed. Throwback rule is an Internal Revenue Service (IRS) rule applied in the taxation of trust property. The rule specifies that the amount distributed in any tax year which is in excess of that year's distributable net income must be thrown back to the preceding year and the amount is treated as if it had been distributed. The throw back rule requires consideration of the trust income and its distribution for all of the years preceding the tax year

Which of the following is NOT a generation-skipping transfer?

- Taxable redemption A direct skip, taxable distribution and taxable termination are generation-skipping transfers. An estate tax is equal to the maximum estate tax rate at the time a taxable distribution, taxable termination or direct skip is made. A taxable redemption is not a generation-skipping transfer.

Which of the following trusts are considered flexible:

- Testamentary trust - Revocable trust Revocable trusts are flexible. The terms of the trust can be amended, altered, or revoked in their entirety by the grantor if the grantor finds that the terms of the trust are not fulfilling the grantor's estate planning objectives. Testamentary trusts are flexible until death since they do not take effect until death. An irrevocable trust is irrevocable and therefore rigid.

Matt suffers from schizophrenia and thinks that his wife Dona is out to kill him. Therefore, in his will he bequeaths all property to his two sons and a daughter, leaving very little to Dona. Dona wants to contest the will. Under what basis should she contest the will?

- That the maker suffered from an insane delusion at the time the will was drafted An insane delusion differs from an unsound state of mind. To be of unsound mind, the Testator must not know who he or she is, or the extent of his or her property ownership, or who the natural objects of his or her affection are, or that he or she is making a will and giving away his or her property. In the case of Matt, he is fully aware of his identity that he is making a will, and giving away his property. He knows that Dona is his wife but disinherits her because he suffers from an insane delusion (delusional schizophrenia) that she is trying to kill him.

The value of Catherine's retained interests was determined at the time of subsequent transfer of interest. If the interest was valued at zero at the time of transfer, then how would the taxable value on the interest she receives be calculated?

- The amount by which Catherine's taxable gift value has increased. The taxable value would be calculated on the amount by which the value of the taxable gift has been increased. It would not be calculated on the original gift, as the tax for the same has been paid by the person who retained it. It would not be calculated on the fair market value of the total taxable gift as that would amount to double taxation

What is a corpus?

- The amount of principal in a trust

The gift tax can be reduced or eliminated by using all of the following techniques?

- The annual exclusion - The applicable credit - Qualified disclaimers - Discounts for lack of marketability and minority interests The GSTT exemption is used to offset any generation skipping transfer taxes that may result from gifting property to individuals who are two or more generations below the transferor. The GST tax is imposed in addition to the gift tax and the estate tax, but the GSTT exemption is not available to offset these taxes.

Who signs the written receipt verifying that beneficiaries who were supposed to receive property from the estate have actually received the property?

- The beneficiaries themselves

Assuming a GST exemption is not allocated to a trust, which party is responsible for paying a GST tax when a distribution is made to a skip person beneficiary?

- The beneficiary The beneficiary is obligated to pay the GST tax in a taxable distribution. The recipient can deduct the GST tax paid on the distribution on his own personal income tax return. The tax is tax inclusive, meaning the distributed amount is reduced by the amount of GST tax the beneficiary must pay.

A blockage discount occurs when:

- The block of stock to be sold is too large to be marketed in an orderly manner - Selling prices and bid and asked prices may not reflect the fair market value A blockage discount occurs when a large block of stock that cannot be marketed in an orderly manner results in depressing the market. If this is the case, selling prices and bid and asked prices may not reflect their fair market value. The converse of the blockage situation is where the block of stock to be valued represents a controlling interest either actual or effective in a going business. Here, the large block could have the effect of increasing value because of its element of control.

Which of the following events can trigger business continuity contingencies?

- The buy-sell agreement specifies the triggering event. Generally that event is death, disability, or retirement of the owner. It can include divorce or insolvency, and events such as loss of a professional license by an owner or conviction of an owner of a crime.

A combination of a non-marital By-pass trust and a QTIP trust would logically be recommended to a wealthy client in all of the following circumstances, EXCEPT:

- The client has confidence in the spouse's ability to manage finances. The combination of nonmarital trust and QTIP trust would be more appropriate for a client who wants to retain control over the assets after death and does NOT have confidence in the spouse's ability to manage assets. The trusts provide an income stream for the surviving spouse, but the surviving spouse is given no control over disposition at death.

Please select which requirements match the legal requirements of a CRUT.

- The donor must make an irrevocable transfer of the property. - The donor can make more than one transfer of property to the trust. - The trust must pay at least 5% of the annually reappraised value of the corpus. - The income payable to trust beneficiaries increases with each year as the corpus increases in value or decreases as the case may be. With a CRUT, the donor can make more than one transfer of property to the trust. The trust must pay at least 5% of the annually reappraised value of the corpus. The income payable to trust beneficiaries in a CRUT increases with each year as the corpus increases in value. The donor is entitled to an immediate income tax deduction for the present value of the property that passes to the charity, as he or she is the remainderman. If the trust is structured to take effect upon death of donor, so that some beneficiary receives the income, then donor's estate receives an estate tax charitable deduction.

Please select which requirements matches the legal requirements of a CRAT?

- The donor must make an irrevocable transfer of the property. - The donor can make only one initial transfer of property to the corpus. - The trust must pay at least 5% of the initial value of the corpus. - The income payable to trust beneficiaries remains fixed once the initial payments are calculated. With a CRAT, the donor can make only one initial transfer of property to the corpus. The trust must pay at least 5% of the initial value of the corpus. The income payable to trust beneficiaries in a CRAT remains fixed once the initial payments are calculated. The donor is entitled to an immediate income tax deduction for the present value of the property that passes to the charity, as he or she is the remainderman. If the trust is structured to take effect upon death of donor, so that some beneficiary receives the income, then donor's estate receives an estate tax charitable deduction.

What is valuation of transferred property?

- The first step in gift tax computation process

Adjusted taxable gifts must not be added to the taxable estate in 2012 if:

- The gift was made before 1976. Adjusted taxable gifts are defined as the taxable portion of all post-1976 gifts. A gift is taxable to the extent it exceeds any allowable annual gift tax exclusion, gift tax marital deduction or gift tax charitable deduction.

The grantor pays the income tax on any income produced by the trust, provided the following conditions occur.

- The grantor indicates that the income is to be used to pay life insurance premiums for a policy owned by the grantor or the grantor's spouse. - The grantor may have the income payable to him, under the terms of the trust. - The grantor has the ability to revoke the trust or alter its terms so that the trust terminates or reverts back to the grantor at any time.

Mary and Doug lived in Massachusetts and bought a house with money Doug received as an inheritance. They sold their home in MA. and moved to California, using the proceeds of the sale to buy a new home in Los Angeles. Which statement correctly describes how the home will be treated at Doug's death?

- The home will be treated as separate property in Doug's estate. The home is treated as separate property in Doug's estate. The house was not marital property at the time of acquisition, because it was acquired by inheritance. Therefore, when moving to CA. the house remains Doug's separate property.

An irrevocable trust owns the life insurance of a shareholder. At his death, the proceeds are paid to the policy beneficiary who then makes a fully secured loan to the corporation. The corporation then redeems the stock under Sec. 303. Which of the following are the advantages of this technique?

- The insurance proceeds don't swell the value of the corporation for estate tax purposes. - Cash values cannot trigger an accumulated earnings tax problem. - Neither cash values nor death proceeds can trigger an alternative minimum tax problem. By using these techniques, the insurance proceeds do not swell the value of the corporation. Furthermore, cash values do not trigger a problem caused by accumulated earnings. Finally, neither cash values nor death proceeds pose an alternative minimum tax problem.

What is presumptive disability?

- The insured is always considered totally disabled, even if he or she is at work, if sickness or injury results in the loss of any body parts.

What is own occupation?

- The insured is totally disabled to perform the major duties of his or her regular occupation.

All of the following choices are typical execution requirement(s) for the maker of a will, except:

- The maker must not be a minor in any case. The maker must either be of age or, if married, a minor who is considered to have reached the age of majority because of the marriage.

Which of the following factors will have the most determining effect on the probative value of sales?

- The market price in an established market When an organized market exists, the market price will prevail. However, the other factors also affect the value considerably. The courts tend to disregard isolated or sporadic sales. It is unusual for sales between parents and children or employers and employees to be given great weight in the light of their almost definitional unequal bargaining positions. Offers to purchase or sell, as opposed to options, present little evidence of value.

Francis, who died in 2012, willed the stocks of International Business Machines (IBM) to his nephews Tony and Sam. The stocks of IBM have an established market and the quotations for these stocks are available at any date in all the stock exchanges. What is the fair market value of the IBM shares for inclusion in Francis's gross estate?

- The mean between the highest and lowest quoted selling price on the valuation date Where a stock has an established market and quotations are available to value the stock as of the date in question, the fair market value per share on the applicable valuation date governs the valuation for estate tax purposes. The fair market value is the mean between the highest and lowest quoted selling price on the valuation date. The opening and closing selling prices do not affect the fair market value.

Carolyn purchased a life annuity with the provision that payments would begin when she reached age 60 and would continue until her death. Carolyn's will states that all of her estate should go to her only child, Dorothy. Carolyn dies at age 82. What happens to the annuity?

- The payments would cease with her death - The value of annuity interest is not includable in gross estate, except for the last payment. The life annuity does not have a provision of continuing payments to her daughter, so the payments would cease with Carolyn's death. Such contracts that provide payments to the decedent and end at death are not subject to the provision of IRC Section 2039 because they are not capable of death time transmission, that is, there is no transferable interest. Therefore, the value of annuity interest is not includable in Carolyn's gross estate, except for amount which would be included in her estate would be the last payment she had been entitled to receive if such payment was due before her death.

Keeping an inventory of all probate assets, payment of creditors' claims, and distribution of the probate estate according to the terms of the decedent's will, or according to the laws of intestacy if the decedent dies without a will, are functions of:

- The personal representative Administration of the probate estate occurs only after the letters testamentary have been issued to the personal representative. The administrative functions of the personal representative are an inventory of all probate assets, payment of creditors' claims, and distribution of the probate estate according to the terms of the decedent's will, or according to the laws of intestacy if the decedent dies without a will.

The decedent has retained the power to alter, amend, revoke, terminate or affect the beneficial enjoyment of property, which can be transferred if what powers are retained?

- The power to change the beneficiaries - The power to hasten or delay the timing within which beneficiary can enjoy or receive property - The power to increase or decrease the property allocated to a beneficiary

What is a health care power of attorney?

- The purpose of the health care power of attorney is to grant a trusted person the power to make health care decisions for you should you become incapacitated. - A health care power of attorney document older than seven years may need to be updated.

Which document contains the sworn statements of the witnesses that the will met all valid execution requirements for the state in which it was executed?

- The self-proving affidavit The self-proving affidavit contains the sworn statements of the witnesses that the will met all valid execution requirements for the state in which it was executed. The other documents do not contain the above statement.

Before a gift tax is applied, it is essential to have a complete transfer. Choose the options that are true regarding complete transfer.

- The subject of gift is put beyond the recall of the donor. - The donor parts with a gift, which is irrevocable. - The amount of the gift cannot be altered. The subject of the gift is put beyond the recall of the donor in complete transfers. Donors part with dominion and control over the gift irrevocably. In a complete gift, the donor does not have the power to change the identity of the donee. The donor cannot alter the amount of the gift.

What qualifying requirements must the QDOT satisfy in order to qualify for the marital deduction?

- trust instrument requires that at least one trustee be either a U.S. citizen or a U.S. domestic corporation - requires no disribution of corpus from the trust may be made unless that trustee has the right to withhold from the distribution the QDOT tax imposed on the distribution - The QDOT election under 2056A(d) has been made for the trust by the executor of the estate on the decedent's estate tax return

Jane's employer purchased an annuity contract in 2007 for Jane when she was 40 years old. The provisions were that when she reached age 58, she would start receiving payments of $10,000 per year. After her death, her son would receive the payments. Jane dies in 2012. Her son starts receiving the payments immediately after her death. Which of the following statements is not true?

- The value of annuity interest is includable because it involves a transfer of property - The value of annuity interest is not includable because Jane did not start receiving them - The value of annuity interest is not includable because Jane did not purchase the annuity contract Under IRC Section 2039, if an annuity is payable as a result of the recipient's surviving the decedent and if the payment is made under a contract that provided the decedent with a payment or a right to a payment for life, the value of that income right is includable in the decedent's gross estate. Hence, the value of annuity interest is includable in Jane's gross estate because she had a right to the payments, and not because it involved transfer of property at death. It is includable regardless of whether she actually received them or not. Furthermore, if the decedent's employer furnished all or part of the purchase price, that contribution is treated as if it were made by the decedent.

Cedric Lodge left a huge estate to his daughter Lucy when he died. The executor of his estate approaches you to advise him about the valuation of Mr. Lodge's estate. What do you think could be the various factors that will affect the valuation of Mr. Lodge's real property? Click all that apply.

- The value of net income received from the property - The size, shape and location of the property - The nature and condition of the property - The actual and potential use of the property

Under IRC Section 2037, what is includable in the gross estate if the donee is alive when the donor dies?

- The value of the property transferred If the donee can obtain possession or enjoyment through ownership only by being alive at the time the donor dies, and the donor retained a significant right to regain the property personally, the value of the property transferred and not the value of the interest retained will be includable. The "significant right" represents a more than 5% chance to regain the property.

David Struthers transferred his estate to his daughter Jennifer in 2009 with the condition that she would receive income until age 35, at which age she would receive the principal, if she were not married. He retained the power to distribute the income to his son, Stephan, if Jennifer should get married. David dies in 2011 when Jennifer is 32 years old and not married. Which of the following is includable in David's gross estate?

- The value of the remaining income term interest for three years from David's date of death to when Jennifer reaches age 35 According to IRC Section 2038, in David's case only the transfer of income is a revocable transfer. Only the value of property subject to this power will be included in the gross estate and not the entire value of the transfer of property. The property for which David retained this power at the time of his death was the remaining income that would be paid to Jennifer until she reaches age 35. David did not retain the right to the transfer of principal because Jennifer would receive it in any case at age 35, and therefore the principal is not includable.

Some reasons clients do not plan an estate inclue:

- They assume that someone else will handle the distribution of their property after their death. - They want to avoid unpleasantness emanating from family problems that may result from their planning. - They are too busy to be bothered about planning their estate. - They are always on the move, so they lack the sense of permanence to plan their estate. People shy away from estate planning either because they don't want to be bothered, do not have time, or assume that someone else will handle the distribution of their property after their death. In some cases people realize that there could be disputes or unpleasantness as a consequence of their planning.

Rebecca Mark's 80-year-old Aunt disowned her children and left behind an estate worth $4 million to Rebecca, her grand-niece. A generation-skipping trust was set up for the benefit of Rebecca, and her Aunt allocated a GSTT exemption of $4 million to the trust. Choose all options regarding generation-skipping trusts that apply in this case. Click all that apply.

- This trust is a direct skip trust. - Rebecca is a skip person beneficiary. The trust is a direct skip trust since there is only one beneficiary who is a skip person. A skip-person includes great-nieces and nephews. Rebecca is not subject to a GST tax on the trust property at her death.

What is a life insurance trust?

- This trust is structured to own and receive either a life insurance policy or the death benefits payable upon the death of the insured.

A business continuation agreement states the purchase price, terms, and funding arrangements. According to the agreement, to which of the following does a retiring owner or his estate have to sell the business?

- To the business itself - To the surviving owners - To a third party nonowner

Tom Sanders is in his mid-seventies and wishes to take appropriate estate planning measures to transfer his estate property in Florida. For GST purposes:

- Tom's wife would be assigned to the same generation as that of Tom. All persons are assigned to a generation for GST purposes. If it is a related person, then the person is assigned by reference to the ancestral chain relating back to the transferor. Spouses of the transferor are assigned to the same generation as the transferor. If an unrelated person is younger than the transferor by 12 ½ years, then he or she will be assigned to the same generation as the transferor. An unrelated person will be assigned one generation below the transferor if he or she is younger than the transferor by 12 ½ to 37 ½ years.

Choose the transfers that are excluded from generation-skipping transfers.

- Transfers made by gift to a donee who is one generation below the donor - Tuition payments made directly to a college and a payment made to a doctor for a grandchild's medical expenses - Transfers already subjected to GST tax, in which the transferee was in a lower generation than the present transferee

Avoiding probate helps in?

- avoiding publicity associated with the probate of the estate, lengthy time delays, and high costs.

What is the benefit of GRIT?

- Transfers property to a beneficiary at a lower gift tax valueWhen you create an irrevocable trust, you forfeit ownership of all assets you put in it. That means you lose control over those assets, and no longer have the right to enjoy them - they've become someone else's property. However, with GRITs, GRATs, and GRUTs, the grantor (the person whose assets go into the trust) gets to maintain some right to trust property. For example, you have a commercial rental property that is worth $8M, and generates tens of thousands of dollars in lease income. If you put the property into a GRIT, you lose ownership interest in the property, but have the ability to keep the rental income stream. GRITs are primarily used to avoid or minimize estate and gift taxes while allowing the grantor the ability to keep an income stream. Taxes can be avoided or minimized because when assets are put into a GRIT, they are no longer part of the grantor's estate, and as such, the value of the estate drops (hopefully below the estate tax threshold). A GRAT is similar to a GRIT in that both allow the grantor to retain some income rights from the assets placed into the trust. In a GRAT, that income is in the form of an annuity. So, if you own $1M worth of stock and receive a 5% dividend every year, and decide to put that stock into a GRAT, you lose ownership of all of your shares but retain the right to receive your annual $50k in the form of an annuity. Again, using type of trust is a great way to avoid or minimize estate taxes. Finally, a GRUT allows the grantor the right to receive income from the assets placed into the trust. However, the income is earned in the form of a percentage of the value of the trust's assets. This perce ntage is determined yearly, at a specified date, for the term of the trust (by "term" I mean the number of years the grantor retains a right to any income, before it goes to the beneficiary).

Henry Ford goes to a financial planner for help. His financial planner calculates the value of his gross estate as an important factor in the selection of his estate planning technique. State True or False.

- True

A codicil is used to revoke all or a portion of a will. State true or false.

- True A will is a revocable instrument. Therefore, it can be amended, altered, or revoked a number of times prior to the testator's death. Many wills are revised by using codicils, which may make changes to all or a portion of the existing will.

Oliver Williams heads a large business group. He decides to plan his estate because he does not want to leave his business to incompetent people. State True or False.

- True Effective estate planning provides for an orderly distribution of business. It helps the client to make provisions to leave his business to an efficient employee or competent family member.

If the donor is making a gift of cash to a public charity, the maximum income tax deduction that may be taken is 50% of the donor's AGI. State True or False.

- True If the donor is making a gift of cash to a public charity, the maximum income tax deduction that may be taken is 50% of the donor's AGI. Keep in mind that the AGI rules must be satisfied in each year a charitable deduction will be taken, even if the deduction is being carried forward.

Life insurance proceeds and disability income used for paying for a stock redemption are not tax deductible to a corporation, but rather are corporate income tax free. State True or False.

- True Insurance proceeds and disability income used for paying for a stock redemption are considered tax free for corporate income.

When selecting an attorney, the client should find the one who charges comparably lower fees. State True or False.

- True Selecting the right attorney is a cost-saving factor. However, the lowest fee may be far from the least expensive because of the complexity of the practice and the potential for making expensive mistakes. Malpractice premiums for estate planning and administration attorneys are among the highest in the profession.

Unpaid cummulative distributions are not counted in the decedent transferor's estate if they are transferred along with the applicable interest to a surviving spouse, and the transfer was full and adequate. State True or False.

- True Taxes are deferred for transfers of unpaid cummulative distribution for spouses because of marital deduction and if the transfer is for full and adequate consideration. The spouse will inherit the tax consequences on a subsequent taxable event.

Estate planning provides for the welfare of a client's minor children. State True or False.

- True The welfare of minor children can be structured and supervised through formal estate planning. Trusts, guardianships, and conservatorships ensure that minor children are provided for.

Tax savings while alive is through which process?

- both income and gift tax savings

Costs involved in the probate of an estate are likely to be higher than if probate is avoided because most administrative and attorney's fees are based on a percentage of the value of the probate estate. State True or False.

- True Therefore, the greater the amount of property that is transferred outside the probate estate, the smaller the amount of administrative and legal expenses incurred by the estate. Additional costs also may be involved if the probate procedure includes appointments of a guardian, conservators, or appraisers.

Lynn is an owner of a closely held business that is appreciating rapidly. She decides to transfer $400,000 of stock to her children who work in the family business to reduce the value of her estate, and have her children benefit from the company's growth. Lynn wants to receive income from the business but she does not structure the income payments as qualified payments. Therefore, her retained business interest is valued at zero. True or False

- True Under Chapter 14, a qualified payment is needed to have the stock transferred to her children valued at $400,000. With a retained business interest of zero, the gifted shares are valued at the full value of Lynn's business interest. If Lynn had recapitalized her stock into non-voting common and cumulative voting preferred shares, and gifted the common stock to her children, she would have "qualified payments" and the common stock would be valued at $400,000 when transferred.

Whose responsibilities commonly last for one or more generations?

- Trustee Unlike the role of the executor, which is typically concluded within a year or two, the trustee's responsibilities commonly last for at least one generation and often last beyond two or three generations. The time span of the duties of the attorney will depend on the client's stage in the estate planning cycle, such as accumulation, conservation or distribution, but usually are within the client's lifetime.

Which of the following is excluded from GST tax?

- Tuition paid to a university for a skip person's education. - Gifts already subjected to GST tax and the new transferee is the previous transferee's son.

Most trusts are required to have terms or conditions under which the trust will terminate or fail. If the trust does not have conditions or terms under which it will terminate, so that the trust could conceivably be of infinite duration, then the trust will violate the rule against perpetuities and will fail:

- Unless the trust beneficiary is a charity If the trust does not have conditions or terms under which it will terminate, so that the trust could conceivably be of infinite duration, then the trust will violate the rule against perpetuities and will fail unless the trust beneficiary is a charity.

What type of trust would a client and spouse with managerial experience use?

- Use a trust in which the spouse or grantor is named as trustee.

What is CRAT?

- Used in situations where the donor wishes to provide a noncharitable beneficiary with a stream of income to last for a stated term of years.

What is charitable stock bailout?

- Used in situations where the owner of closely held stock wishes to make a gift of property to a qualified charity.

What is CRUT?

- Used when the donor wishes to have a variable amount of income distributed to the income beneficiary.

What is so-called blockage rule?

- Valuation of stock may be lower than its current FMV

Patrick is doing research on trusts. He is currently studying the reasons why a trust may fail.

- Violates the rule against perpetuities - Illegal purpose of the trust - Termination by agreement of the beneficiaries Trusts that violate the rule against perpetuities fail because the trust cannot be perpetual but can only last for a period of time that is measured by the lives of any persons alive at the time the trust is created. A trust with an illegal purpose fails because if the trustee or any of the beneficiaries commit an illegal act, the trust is illegal and terminates on the occurrence of the act deemed illegal. Termination by agreement of the beneficiaries occurs if all the beneficiaries agree to terminate the trust provided the purpose of the trust is achieved. Multiple trustees and multiple beneficiaries will not cause a trust to be terminated.

Ronald, the executor to the owner of Wells Inc. is in a meeting with his financial planner Margaret Plummer. He would like to know whether some of the stock can be redeemed as Sec. 303 stock redemption to pay for funeral expenses and estate taxes in 2012. Margaret explains the situations where he can use a Sec. 303 stock redemption. In which of the following situations can he use Sec. 303?

- When the corporation's stock is a major estate asset and a forced sale or liquidation of the business in order to pay death taxes and other costs is a threat. - Where a tax-favored withdrawal of funds from the corporation at the death of the stockholder would be useful. - When redemption of IRC Sec. 303 stock is desirable. Ronald can use the Sec. 303 redemption to take care of death taxes and other costs which could possibly cause liquidation. The other situations where this can be used are when it would be useful to withdraw funds from the corporation on the death of the stockholder and when redemption is desirable.

In the case of a decedent dying with a will, the will is filed with the probate court. Generally, in what county does this occur?

- Where the decedent resided at the time of death In the case of a decedent dying with a will, the will is filed with the probate court. Generally, this occurs in the county where the decedent resided at the time of death. If the decedent dies without a will, no document is filed.

To read the case study for this question, click here. In order to effectively disclaim property for federal transfer tax purposes, Annette must make the disclaimer:

- Within nine months of receiving the property. For a disclaimer to be effective, it must be received by the transferor, his legal representative, or the holder of legal title to the property no later than nine months after the day on which the property is received. In the case of a minor, it must be received within nine months of his or her attaining the age of 21.

Kirk has a problem. He has a disabled wife who has suffered disability through an accident. He wants to arrange a trust in such a way that assistance to his wife does not disqualify her from receiving public assistance benefits and Medicare. He wants to know whether such an arrangement can be done. What would you tell Kirk?

- Yes. It can be done. Assistance to a family member who has suffered disability through an accident (or who has been disabled since birth or childhood) can be done by arranging a trust in such a way that it enables the disabled beneficiary to receive assistance from the trust. These trusts are known as special needs trusts. And such an arrangement can be done in such a way that receiving assistance from the trust does not disqualify the disabled person from receiving public assistance benefits, Medicare, or other forms of supplemental assistance.

What are the basic means of qualifying a transfer to minors under Section 2503?

- a Section 2503(b) trust - a Section 2503(c) trust - the Uniform Gifts to Minors Act There are three basic means of qualifying "cared-for gifts" to minors under Section 2503: (1) a Section 2503(b) trust, (2) a Section 2503(c) trust, or (3) the Uniform Gifts to Minors Act (or the Uniform Transfers to Minors Act). Section 7520 pertains to rates of gift tax.

To determine the taxable estate, allowable marital and charitable deductions are subtracted from the:

- adjusted gross estate The taxable estate is determined by subtracting from the adjusted gross estate any allowable marital deduction or charitable deduction. Tentative tax base is used in calculation of estate tax payable before credits.1.Determine the value of the gross estate. 2.Subtract allowable deductions. 3.Add the value of taxable gifts made after 1976 if not already included in the gross estate. (Remember that the estate and gift tax rate is based on cumulative transfers. The gift tax paid on taxable gifts made after 1976 is subtracted in Step 5.) 4.Consult tax tables for the unified rate schedule for the applicable rate of tax and apply the unified estate and gift tax rates. 5.Subtract the amount of gift taxes paid on taxable gifts made after 1976. 6.Subtract the allowable unified credit and any other allowable credits. The resulting amount is the net federal estate tax owed by the estate.

To qualify for social security disability benefits on the wage earner's record, a spouse must be?

- age 62 or older - caring for a child who is disabled or under 16 - 50 or older and disabled before the wage earner's death or within seven years after death

Herbert was in the process of drafting his will and was keen on minimizing estate tax in 2012. He found out that the unlimited marital deduction would allow:

- an entire estate to be transferred to the surviving spouse without estate tax. The maximum amount allowable as a marital deduction for federal estate tax purposes is the net value of the property passing to the surviving spouse in a qualifying manner. Otherwise, there is no limit to the marital deduction. An individual could conceivably transfer his or her entire estate to the surviving spouse estate tax-free.

The general power of appointment exists when:

- an individual has a right to dispose of the property that he or she does not own - an individual can exercise it in favor of self, creditors, his or her estate or creditors of the estate A general power of appointment exists when an individual has the right to dispose of property that he or she does not own, including giving it to himself or herself. It includes the power to appoint to the powerholder, the powerholder's estate or the creditors of either. A special or limited power of appointment enables the individual to appoint anyone other than self, his or her estate or creditors to receive property.

The administrative functions of a personal representative are:

- an inventory of all probate assets - payment of creditors' claims - distribution of the probate estate

The tax credits that can be applied to the estate tax for determining net federal estate tax are:

- applicable or unified credit - credit for foreign death taxes The unified credit, credit for foreign death taxes and credit for tax paid on prior transfers are applied to the estate tax for determining net federal estate tax. The child tax credit, lifetime learning credit and dependent care credit are not relevant in the context of federal estate tax. There is a state death tax deduction, not a credit, available that is subtracted from the adjusted taxable estate.

Which of the following are probate fees:

- appraisal fees for probate assets - court filing fees - attorney's fees Payment of creditors' claims and distribution of estate property to beneficiaries are a part of the probate process. They are not additional expenses incurred by the estate as part of the probate process. Probate fees are any additional expenses incurred by the estate as a part of the probate process. Probate fees include appraisal fees for probate assets, court filing fees, attorney's fees, and other administrative expenses.

What is a non-marital or bypass trust?

- appropriate when the grantor wishes to leave a life income interest to his spouse as well as other family members such as children or grandchildren.

What is laws of intestacy?

- are those state laws which determine how a decedent's property is distributed in the absence of a will.

Beth is a five-year-old child. She inherits a trust property that is structured so that she has the right to claim a maximum amount under a five and five power that is 5% of the value of the corpus.

- crummery trust If a trust is properly structured a minor beneficiary can demand the lesser of the amount transferred annually to the trust or the maximum amount allowed under a five and five power which works out to be the greater of $5,000 or 5% of the value of the corpus. In such a case the minor beneficiary has a right, better known as a Crummey right. Such a trust is termed a Crummey trust.

Who is the ability to make financial decisions?

- durable power of attorney - plenary guardian

Tax savings at death is through which process?

- estate tax savings

What are the federal estate taxes based on?

- fair market value of the transferred property as of the date the decedent died - the value of the property six months after the date of the decendent's death, which is alternative valuation date

What is the maximum annual charitable income tax deduction for future interests gifts of property?

- generally do not qualify for the charitable contribution deduction.

The unified credit or applicable credit is used as an offset against:

- gift tax - estate tax The term unified credit is used because under the unified transfer tax system the same credit amount is used to offset gift taxes and estate taxes. This credit does not offset the income tax or the inheritance tax.

Funeral expenses, subject to certain limitations, are deductible. Deductions are generally limited to a reasonable amount. Such expenses would include:

- interment - burial lot or vault - grave marker - perpetual care of the grave site - transportation

What is execution requirements?

- is a legal instrument which makes changes to all or a portion of an existing will and must be executed with the same execution formalities as a will.

What is noncupative will?

- is a verbal (oral) will that bequeaths the decedent's personal property

What is reciprocal will?

- is a will executed by two people who agree to leave their property to each other

What is holographic will?

- is a will written in the handwriting of the maker

A springing durable power of attorney:

- is activated when the principal is declared incompetent A springing durable power of attorney is activated when an agent receives notification from a physician that the principal is found to be mentally or physically incompetent.

What is joint will?

- is one document that serves as the last will and testament for two individuals

What is revocable?

- is something which can be amended, altered, or revoked a number of times prior to the maker's death.

What is a testator?

- is the property owner who drafts the will.

What are the advantages of using a durable power of attorney vs. a conservatorship for managing an incompetent person's property?

- it avoids a public court hearing to declare incompetency - it avoids the need for court supervision and ongoing legal fees - the principal rather than the courts can appoint a fiduciary - the principal retains title to the property Many lending institutions will not accept all durable powers of attorney, and others will accept them only if they are current and meet the institution's guidelines.

What happens with a testate estates?

- left behind by a decedent with a will

What happens with a intestate estates?

- left behind by a decedent without a will

How can beneficiary show competence?

- legal - mental - financial

What are five typical execution requirements for the maker of a will?

- maker must be of sound mind when executing the will - maker must be age of majority - maker, also known as the testator, must sign the will - will must be declared as the last will and testament of the testator - will must be witnessed by at least two competent individuals

What is the maximum annual charitable income tax deduction for gifts of long-term capital gain property?

- maximum annual charitable deduction is 30% of the donor's AGI. When the donor deducts only the basis, he is eligible for a 50% deduction on 50% of his AGI.

What is the maximum annual charitable income tax deduction for gifts of ordinary income property?

- maximum annual charitable deduction is 50% of the donor's AGI, based upon the donor's basis, except where the donor is limited to his basis in the property.

What is the maximum annual charitable income tax deduction for gifts of cash?

- maximum annual charitable income tax deduction is 50% of the donor's AGI.

Revocable living trust may not be?

- may not be in writing.

How does the special use valuation reduce estate taxes?

- may reduce the value of qualified real estate in the owner's estate

A testamentary trust must be?

- must be in writing.

How does the alternate valuation date reduce estate taxes?

- net reduction in gross estate six months after death

An insane delusion differs from a state of unsound mind. State True or False.

- true To be of unsound mind, the maker must not know who the maker is, or the extent of his or her property ownership, or who the natural objects of his or her affection are, or that he or she is making a will and giving away his or her property. An insane delusion, on the other hand, may be a situation in which the maker knows who his or her heirs or family members are but disinherits them because he suffers from an insane delusion that they are trying to kill him.

Jim and Janet have lived together for four years. Jim owns several storage units that he rents, but he does not need the income from them currently. He has a son Eric that he wants to give some income to for the next 5 years, and he wants Janet to receive some of the income for life. He also wants Eric and Janet to each own ½ of his storage units at his death. He is thinking of converting his units to a JTWROS with Janet. Will this form of property ownership meet his goals?

- no If Jim converts his property to JTWROS with Janet he is making a gift of ½ of the property. She would be able to receive income from her ½ share, and Jim could gift his ½ share of the income to Eric for the next 5 years. However, he could not bequeath ½ of the units to his son at death since they would pass automatically to Janet. Instead, Jim could convert his units to a tenancy in common with Janet that would provide her with ½ ownership and ½ of the income for life. He could gift his income to Eric for the next 5 years and bequeath his ½ share of his property to Eric at his death.

Mark passed away in January 2012, leaving behind a vast estate and a will. His nephew Frank was left with just $100,000. Frank was given a written notice after the will was admitted to probate in February 2012. The time period for any interested party to contest the probate was stated as six months from the date of admission of the will to probate. Frank felt that the amount given to him was much less than that given to others and therefore humiliating. He consulted his lawyers and finally filed a case to contest the probate in September 2012. Will he get more money?

- no Once an interested party (in this case, Frank) has been given notice and fails to contest the probate process within the specified period of time, that is six months from the date of admission of the will to probate, the interested party (Frank) is barred from raising any further challenges to the probate of the will. So even if his case was valid and he deserved more from the will, he couldn't challenge the probate of the will, as he filed his case to contest the probate after the specified time period

Kirk passed away in 2012, leaving his entire estate to his wife Sharon through a marital trust. He had structured the marital trust in such a way as to provide a stream of income to the surviving spouse for the spouse's lifetime. Kirk's executor qualified the property for the marital deduction in his estate. Sharon believes that the transferred property will not be included in her gross estate when she dies, as the property qualified for a marital deduction in Kirk's estate. Is she right in her thinking?

- no The property placed in the QTIP marital trust qualifies for the marital deduction in the gross estate of the decedent (Kirk). Therefore, although it is included in the decedent's estate, it is not subject to estate tax. Assets left within the trust at Sharon's death will be included in the gross estate of the surviving spouse (Sharon).

Identify which organizations are qualified public charities from the list below.

- nonprofit schools - universities - synagogues - YMCA As long as the organization has, as its primary purpose, some religious, educational, philanthropic, scientific, or literary purpose, and is intended to benefit the public at large, it most likely will meet the definition of a qualified public charity. Qualified public charities include nonprofit schools, universities, institutions of higher learning, churches, synagogues, the YMCA, the YWCA, the United Fund, the United Way, the American Red Cross and the Boy Scouts and Girl Scouts of America. Qualified organizations also include groups whose primary purpose is to assist in the discovery of a cure for a disease, that is, organizations such as the Heart Association, the American Cancer Society, the American Diabetes Association, the Arthritis Foundation, and the Society for the Prevention of Blindness. Most political action committees and political action groups (for example, the National Rifle Association, or any political group that hires lobbyists, such as the American Dairy Association) do not qualify for a charitable deduction, if their purpose is to influence legislation or political ideologies.

A surviving spouse cannot effectively use a disclaimer for federal transfer tax purposes to:

- pass property to a trust in which the spouse has the power to direct distributions to other beneficiaries In general, no disclaimant can dictate what happens to the property except when the property passes to a trust in which the spouse has the power to direct distributions to other beneficiaries.

What are features of professional trustee?

- permanency - financial security - avoiding conflicts of interest - accounting and tax-planning expertise

Who is the ability to make personal care decisions?

- plenary guardian Plenary guardianship is a guardianship in which the court gives the guardian the power to exercise all legal rights and duties on behalf of a ward, after the court makes a finding of incapacity. A court may appoint a plenary guardian only upon a finding that the ward lacks the capacity to perform all of the tasks necessary to care for his/her person or property and is in need of plenary guardianship services. - health care power of attorney - living will

Special Needs Trusts are created to:

- preserve eligibility for public assistance programs Special Needs Trusts are structured to provide services to disabled individuals while preserving their eligibility for public assistance programs.

What happens with a decedent dying without a will?

- probate administration is initiated by filing a petition for the appointment of a personal representative or other fiduciary

What are three other major areas of focus for the probate process?

- probate of estates and the supervision of trusts - guardianships and conservatorships - commitment of the mentally ill

Select the definition of the term "bequest" from the alternatives given below.

- property transferred pursuant to the terms of the will

What is a marital trust?

- provides the decedent's spouse with control over the decedent's property. The amount placed avoids estate tax liability of the first decedent spouse.

How does charitable deduction reduce estate taxes?

- reduces the adjusted gross estate

How unified credit reduce estate taxes?

- reduces the estate tax liability

To qualify for social security disability benefits on the wage earner's record, an unmarried child must be?

- under 18 - under 19 and full-time high school student - 18 or older with a disability started before age 22

What is a QTIP?

- used when the grantor provides income to a spouse for her lifetime, but passes the remainder interest to someone else.

Rebecca was the only child of Joseph Stern and his wife Elizabeth. When he was 45 years old, Joseph transferred his property to his wife Elizabeth for life. If he survived his wife the property would return to him, if not it would go to Rebecca. The property is includable in his gross estate because Joseph has a:

- reversionary interest in the property Under IRC Section 2037, property is includable in gross estate if the lifetime gift that the decedent made is contingent on the donee surviving the decedent. For Rebecca to enjoy the estate she must survive her father Joseph. If she is not alive at the time Joseph dies, neither she nor her heirs will receive any interest in the property. Joseph retains a reversionary interest, that is, the right to regain the property, under the original transfer. There is no question of joint interest here because Joseph is the sole owner of the property before transferring it to Elizabeth, who then becomes the sole owner of the property. It is Rebecca who has a remainder interest in the property.

What are features of nonprofessional trustee?

- sensitivity to beneficiaries - fees

Having probate helps in?

- settlement of beneficiaries' disputes judiciously in the probate court.

Which of the conditions given below for a witness to sign a will is/are correct? The witnesses must:

- sign the will at the request of the maker - sign in the presence of the maker - sign as a witness in the presence of each of the other witnesses

What are the most common types of wills?

- simple wills - holographic wills - pour-over wills - joint wills - tax-effective wills

What are some of the elements the power of attorney should include?

- specific as to which aspects of the principal's affairs it covers - include a provision authorizing the agent to make elections with respect to retirement plan assets - include a provision dealing with making annual exclusion or lifetime gifts - should give the agent authority to transfer assets into a trus created by the principal

Purpose of a trust?

- tax savings

What are the characteristics of interest-free loan?

- taxable gift

Adjusted taxable gifts are added to the taxable estate to arrive at:

- tentative tax base The tentative tax base is calculated by adding to the taxable estate any adjusted taxable gifts. The tax rate schedule is then applied to determine the tentative tax.

What is the trust property?

- the corpus, the principal or the res.

Health care agents can confer with medical professionals to make decisions concerning:

- the extent of medical treatment that should be provided - where an individual should receive treatment - whether surgery should be performed - what if any medications should be administered

What is the legal title holder?

- the party (the trustee) who has a fiduciary obligation to the beneficiaries.

Property considerations that influence the selection of estate planning techniques include whether:

- the property is difficult to value and divide - the property has a high value - the property is liquid and has a market in the event it must be sold - the property is appreciating or depreciating in value - the death of the client-owner impairs the value of the property All of these considerations play an important part in the selection of an estate planning technique. There are estate planning techniques associated with each type of property. For example, if a property has a high value, and the value is anticipated to go higher or remain the same, and/or has a low basis, many estate planning techniques such as remove the asset from the estate, provide the owner of the property with income for a period of time, and/or make charitable contributions.

Thomas was a rich landowner. Upon his death, Thomas's will failed to comply with all of the state's execution requirements. What will happen to the assets owned by Thomas at his death?

- the property will be distributed according to the laws of intestate succession. Failure to comply with all of the execution requirements causes the will to be treated as an invalid instrument. Therefore, the property distributions contained within the will are null and void. As a result, the state's laws of intestacy will determine how and to whom Thomas's property will be distributed

What are two disadvantages of a joint will?

- the surviving spouse may not be able to change the terms of the will - first spouse's property interests are often construed as being terminable interests and therefore will not qualify for the marital deduction

A dower is the right of:

- the surviving wife to an interest for life in all of the real property owned by her husband during their marriage. Under the common law system, marriage creates a spousal interest in the real property owned by the decedent. Dower is the right of the surviving wife to an interest, for life, in all of her husband's real property. The right of the husband to an interest for life in all of the real property owned by his wife during their marriage is called curtesy.

The IRC Section 2701 valuation rules for transfers of interests in corporations and partnerships would NOT apply to a transfer to:

- the transferor's brother

What happens with a decedent dying with a will?

- the will is filed with the probate court

A disclaimer cannot be used effectively for federal transfer tax purposes to:

- transfer property under the direction of the disclaimant Disclaimants cannot give direction as to where the property will be transferred once they have refused it.

Merrill had set up an irrevocable trust some time in 2006. She passed away in December 2012. In January 2008, she had transferred some life insurance policies owned by her to the trust. Will the assets placed into her trust escape inclusion in Merrill's gross estate?

- yes The primary advantage of irrevocable trusts is that as long as the grantor establishes an irrevocable trust, retains no incidents of ownership over the property, and retains no powers over the corpus of the trust that could be construed as ownership, then the assets placed into such a trust will escape inclusion in the gross estate of the Grantor. The exceptions to this rule do not come into the picture as Merrill transferred life insurance policies owned by her to the trust before three years of her death—only certain transfers (gifts) fall within the three year date of death rule—all gifts are not subject to this rule.

How is the estate treated with JTWROS with spouses?

- ½ value of the property is included in a JT's estate regardless of contribution

An executor may be granted a reasonable cause extension to pay the estate tax after the 9 month payment date, for how many months?

12 months It provides that an executor or administrator can request that the IRS grant an extension of time for paying the tax, up to 12 months from the date fixed for the payment, if there is reasonable cause. Furthermore, at its discretion, the IRS could grant a series of extensions upon the executor's showing of reasonable cause. This series of extensions could, in total, run as long as 10 years from the due date of the original return.

Once an interested party has been given notice and fails to contest the probate process within _______ months from the date of the last publication that the will has been admitted to probate, the interested party is barred from raising any further challenges to the probate of the will.

6

What are the three basic execution requirements of a will, regardless of the state of residence?

1. Testator must be 18. The testator is the person who is creating the will 2. The testator must have mental capacity 3. The will must be in writing, signed by the testator, and witnessed by at least two witnesses. Most states require at least two witnesses

What is the minimum value that can be assigned to a transfer of junior equity interest if the total value of the equity interest is $10,000,000 and the total indebtedness is $5,000,000?

10% (10,000,000) + 5,000,000 = 6,000,000

The 50-50 rule applies to both real and personal property regardless of how it was acquired or when it was purchased. However, it can be used only in the case of a joint tenancy between spouses or a tenancy by the entirety and if the joint interest was created after what year?

1976 The 50-50 rule provides that only 50% of certain property titled and held jointly by the decedent and spouse with rights of survivorship, or as tenants by the entirety, will be includable in the decedent's estate. This is regardless of the size of his or her contribution. This 50-50 rule applies to both real and personal property regardless of how it was acquired or when it was purchased. However, it can be used only in the case of a joint tenancy between spouses or a tenancy by the entirety and if the joint interest was created after 1976.

Select from the following list below the correct sequence to describe a charitable stock bailout. 1.The stock is returned to the corporation, and it allows the donor stockholder to retain corporate control. 2.Mr. Cage (owner of closely held stock) wishes to make a gift of property to a qualified charity. 3.He donates the property to the charity. 4.The charity actually cannot use the stock as it is and would prefer the cash rather than the stock. 5.The charity now has the cash it wanted, and the donor receives a valuable charitable contribution deduction measured by the FMV of the donated stock. 6.The charity receives the stock. 7.The charity returns the stock to the corporation in the form of a redemption for cash.

2,3,6,4,7,1,5 Mr. Cage (owner of closely held stock) wishes to make a gift of property to a qualified charity. He donates the property to the charity. The charity receives the stock. The charity actually cannot use the stock as it is and would prefer the cash rather than the stock. The charity returns the stock to the corporation in the form of a redemption for cash. The stock is returned to the corporation, and it allows the donor stockholder to retain corporate control. The charity now has the cash it wanted, and the donor receives a valuable charitable contribution deduction measured by the FMV of the donated stock.

For how many years does a charitable lead trust pay a fixed income stream to a qualified ?

20 years A charitable lead trust pays a fixed income stream to a qualified charity, usually for a period not exceeding 20 years.

The following seven steps describe the estate planning process. Specify the correct sequence of the steps. 1. Establish and prioritize estate planning objectives. 2. Identify estate planning weaknesses before selecting a technique. 3. Gather significant data from the client. 4. Select an appropriate estate planning technique. 5. Identify the factors that limit or affect the selection of estate planning techniques. 6. Implement the estate planning technique. 7. Monitor the plan for revisions and modifications.

3, 1, 5, 2, 4, 6, 7 The following are the steps in the estate planning process: 1. Gather significant data from the client. 2. Establish and prioritize estate planning objectives. 3. Identify the factors that limit or affect the selection of estate planning techniques. 4. Identify estate planning weaknesses before selecting a technique. 5. Select an appropriate estate planning technique. 6. Implement the estate planning technique. 7. Monitor the plan for revisions and modifications.

Hank already used up his GST exemption so he could not allocate an exemption to the testamentary trust. Assume that during the daughter's lifetime there is no appreciation in the trust. At the daughter's death a GST tax is levied at 35% for the taxable termination. How much was the total tax paid on the original $10 million?

5,762,520 1. 10,000,000 - (10,000,000 x 0.35) = 6,519,200 2. 6,519,200 x 0.35 = 2,281,720 3. 6,519,200 - 2,281,720 = 3,480,800 4. 3,480,800 + 2,281,720 = 5,762,520

If Stan and his wife, Shelley, purchase 100 shares of AT&T for $50,000 and hold the property as joint tenants with rights of survivorship, even if the entire contribution was made by Stan from his salary, what percentage would be included in his estate?

50% Only 50% will be included in his estate. If the stock is worth $120,000 at that time, $60,000 will be included in his estate.-------------------------------------------- Whether the ownership share is included in the decedent's gross estate depends on whether: a) it was owned JTWROS with a spouse, or b) it was owned JTWROS with a non-spouse. If the asset is titled JTWROS or tenancy by the entirety with a spouse, then only 50% of the value will be included in the decedent's gross estate. If the asset is titled JTWROS with a non-spouse, then 100% of the value is included in the decedent's gross estate. However, if it can be proven that the other owner(s) actually contributed to the purchase of the property, then only the percentage contributed is included in decedent's gross estate. Read more: http://www.investopedia.com/exam-guide/cfp/property-titling-transfer/cfp4.asp#ixzz3Wm0bKuSN Follow us: @Investopedia on Twitter

If the additional tax is imposed, it is due _____ months after the date of the disposition of the property or the cessation of its use as a farm or as part of the closely held business. The qualified heir who received such property is personally liable for the additional tax imposed

6

In both testate and intestate estates, all creditors of the estate have an opportunity to file their claims with the probate court. Generally, what is the stated time period a creditor has to file their claim before the claim is barred?

6 months

The monitoring process involves a periodic review. How often should the monitoring process take place?

6 to 12 months

Hank has an estate over $15 million and is in a 35% federal estate tax bracket. He establishes a testamentary trust in 2011. Assume Hank has previously used his GST exemption. If Hank dies in 2012 and leaves $10 million in the trust, how much is left for his daughter?

6,519,200 10,000,000 - (10,000,000 x 0.35)

What is the maximum percentage of income that an insurer will cover so that the total of all monthly indemnity is not exceeded?

70 percent Insurers limit the amount of disability income coverage they will sell to an applicant so that the total of all monthly indemnity does not exceed about 70 percent of earned income for disabled individuals with low annual incomes, grading downward to about 50-60 percent or less for those in the highest income brackets.

Which of the following trusts ensures the payment of federal estate taxes upon distribution of principal?

A Qualified Domestic Trust. A function of a QDT is to allow the U.S. Treasury to collect estate taxes. This is not a factor in other types of trusts.

All of the following examples are special powers of appointment, EXCEPT:

A power exercisable in favor of the holder for the holder's comfort. A power exercisable in favor of the holder for the holder's comfort is a general power of appointment. Comfort is too indefinite to provide an ascertainable standard or limitation on expenditure.

All the following statements concerning revocable trusts are correct EXCEPT:

A revocable trust will not remove assets from probate unless the beneficiary receives a vested right to income during the grantor's lifetime. A revocable trust conveys a present, equitable interest in assets to its beneficiaries. The grantor may retain a life interest or a right to income for life, but the revocable trust remains an effective Will substitute even without the beneficiary receiving any income during the grantor's lifetime. The entire value of the trust assets will be included in the gross estate for federal estate tax purposes, regardless of whether the grantor receives a life interest or not.

What type of trust becomes funded when the grantor becomes incapacitated?

A standby trust. A characteristic of a standby trust is that it functions in the event a person becomes incapacitated.

All of the following statements concerning a self-cancelling installment note (SCIN) are correct, EXCEPT:

Any unreported capital gain does not need to be recognized when the note is cancelled or at the owner's death. Capital gains are recognized when the note is cancelled or at the owner's death.

Please review the Case Information: The Unsers Assume for purposes of this question that Samuel Unser's Will provides that Claudia is to receive all income from the trust and she can invade principal to the extent needed for her health, maintenance, and support. Which of the following statements concerning this trust is correct?

At Claudia's death, no amount attributable to the trust or her power of appointment will be includible in her gross estate.

All of the following statements describe the advantages of the probate process, EXCEPT:

Avoidance of publicity because the process is conducted behind closed doors. The probate process is conducted in law courts in a public administration, not behind closed doors.

Please review the Case Information: The Unsers Samuel is considering a buy-sell agreement with the other two owners of Sure Alert Alarm Company. Which of the following statements concerning the use of a buy-sell agreement by the three owners of this company is correct?

If the executor is required to sell Samuel's stock under the terms of a buy-sell agreement, the estate will not qualify for a Sec. 303 redemption.

Please review the Case Information: The Unsers Which of the following statements concerning potential problems with Claudia Unser's estate plan is correct?

If Claudia dies today, the trust provision concerning income for Bryan's issue will violate the rule against perpetuities.

Please review the Case Information: Ann Lytle Assume Ann transfers her personal residence to a qualified personal residence trust (QPRT) with her three children as beneficiaries and retains a right to live in her home for ten years. Which statement correctly describes the consequence of this transfer?

If Ann survives the ten-year term, ownership of the home will pass to her children without any additional gift tax.

How does a client avoid probate?

If the client has property that passes to a named beneficiary outside the will, such as life insurance and IRA accounts, the property avoids probate and results in a reduction of expenses.

Please review the Case Information: The Unsers Assume for purposes of this question that Samuel Unser decided to make a gift of his stock in Sure Alert Alarm Company to an irrevocable trust for his niece Sally, who is 12 years old. Which of the following statements concerning this gift is correct?

If the stock pays no dividends and the trustee cannot sell it, the gift will not qualify for the gift tax annual exclusion.

What is the de minimis rule?

In a lawsuit, a court applies the de minimis doctrine to avoid the resolution of trivial matters that are not worthy of judicial scrutiny. Its application sometimes results in the dismissal of an action, particularly when the only redress sought is for a nominal sum, such as one dollar. Appellate courts also use the de minimis doctrine when appropriate.- Instituted primarily to avoid the bother of administrative record keeping.

In 2006 John bought a life insurance policy on his life with a death benefit of $2,000,000, and named his wife Lucy as beneficiary. In 2011 he created an irrevocable life insurance trust and transferred all incidents of ownership in the policy to the trust. The trust is named the beneficiary of the policy but Lucy has a general power of appointment over the trust corpus. John died in 2012 and the replacement cost value of the policy was $600,000. All of the following statements are correct, EXCEPT:

John's gross estate will include the policy's replacement cost value since John is the insured and the trust is the new owner. The death benefit amount is included in the gross estate due to the 3-year rule.

Please review the Case Information: Ann Lytle Which one of the following assets will be subject to probate after Martin's death?

Personal assets

John Paris made taxable gifts of $15,000 in 2010, 2011 and 2012. Which of the following statements is correct?

Since the gift tax on $45,000 is $9,400 his remaining unified credit amount is 1,763,400. The remaining unified credit amount is calculated by finding the tax on the total taxable gifts- in this case 9,400 and subtracting it from the full unified credit of 1,772,800. That means that John has a unified credit amount of 1,763,400 remaining to offset future taxable gifts.

Carol O'Neil, age 61, has a profitable graphic arts business valued at $800,000 and personal property worth $250,000. Carol wants to retire from her business but she needs $3,000 per month in income. Her neice Sharon, a recent art school graduate, would like to buy Carol's business, but she doesn't have enough money to use as a down payment. What is the best technique that Carol could use to transfer the business to Sharon?

Structure the transfer of the business to Sharon as an installment sale. Carol should set up an installment sale since no down payment is needed and payments to Carol are made from the profits. The payments are secured, and if the PV of any remaining installment payments are brought back into Carol's estate, they won't cause an estate tax.

Albert Soren is the sole owner of an operating farm. Soren has brought his sons into the farm operation, and they now manage much of the operation. The farm has appreciated in the past ten years to over $2.5 million, and the land by itself is worth at least $1.8 million. The farm will be approximately 75% of Soren's gross estate. Soren also owns 500 shares in Midwestern Feed Store, Inc., a closely held corporation. The 500 shares represent a 25% interest in the business. The business is valued at approximately $400,000, but there have been no transactions in the stock and no market for the stock. The farm land would qualify for special use valuation for all of the following reasons, EXCEPT:

The farm operation is the highest and best use of the land. The special-use valuation is only applicable where the use of the real estate is not the highest and best use. The special-use valuation permits a reduction in value where the actual use is not the highest use.

All of the following statements concerning the application of the generation-skipping transfer tax to testamentary direct-skip transfers are correct, EXCEPT:

The federal estate tax is not imposed where the generation-skipping transfer tax is applicable. The generation-skipping transfer tax is imposed in addition to the federal estate tax.

Please review the Case Information: Ann Lytle If Ann makes gifts to each of her 6 grandchildren in 2012, which statement correctly describes the generation-skipping transfer tax (GSTT) consequences?

The gift tax and generation-skipping transfer tax (GSTT) may be applied to the same inter vivos gift.

A husband gives his wife a life estate in his Italian villa valued at $2 million. All of the following statements are correct, EXCEPT:

The husband cannot take an annual exclusion to reduce the gift tax amount of $2 million. The husband can only take an annual exclusion not a marital deduction for this terminable interest property

A QTIP trust may be used to qualify property for the marital deduction. All of the following statements concerning the requirements that must be met for an effective QTIP trust are correct, EXCEPT:

The surviving spouse must be given a general power of appointment over the principal at the time of death. The surviving spouse may be given a limited power, such as the right to distribute the remaining assets of the trust among the children of the marriage.

A QTIP trust may be used to qualify property for the marital deduction. All the following statements concerning the requirements that must be met for an effective QTIP trust are correct EXCEPT:

The surviving spouse must be given a general power of appointment over the principal at the time of death. The surviving spouse may be given a limited power, such as the right to distribute the remaining assets of the trust among the children of the marriage.

The Pastinos own a popular bakery in the neighborhood. Their daughter, Maria, is a divorced mother of two children who is struggling to make ends meet. The Pastinos would like to remove the value of their building and land from their estate while continuing to operate the bakery. They also wish to give Maria $20,000 a year in additional support, however they are "cash poor" since all of their wealth is tied into their business. What estate planning strategy would best accomplish their goals?

Transfer their building and land into an irrevocable trust with Maria as beneficiary, and transfer periodic lease payments into the trust for the bakery's use of the building and the land, to distribute to Maria. A gift lease-back would remove the building and land from the Pastinos' estate, along with any future appreciation on those assets. The bakery would lease back the land and building, which would provide cash to Maria from the lease payments. Payments would be taxed in her lower tax bracket and the Postinos would receive a deduction for the lease payments, which are a business expense. If the Pastinos sold the ovens, they could not operate their bakery.

Please review the Case Information: Ann Lytle If Ann decides to make gifts to each of her 6 grandchildren, which one of the following statements is NOT true?

Transfers to trusts for the benefit of the grandchildren that will only be available to a grandchild upon reaching age 21 will not qualify for the $13,000 exclusion because it is a gift of a future interest.

All of the following statements concerning the use of 2503(b) and 2503(c) trusts (both with a minor as the beneficiary) are correct, EXCEPT:

Under a 2503(b) trust, the trust corpus must pass to the minor beneficiary by age 21. Under a 2503(c) trust (but not under a 2503(b) trust), the trust corpus must be available to the minor beneficiary by age 21.

All of the following methods are used to handle the affairs of an incompetent person, EXCEPT:

Written instructions. Written instructions is the answer. Written instructions is an incorrect example. guardianship, power of attorney, trusts, and living will are all examples of methods of handling an incompetent person's affairs.

Which of the following wills may be entirely handwritten and need not be witnessed but may be effective for disposing of the testator's property?

a holographic will A holographic will is entirely in the handwriting of the decedent and signed and dated by the decedent, but it need not be witnessed.

Which of the following is an important purpose of electing QTIP treatment for estate property?

to assure qualification of some estate assets for the estate tax marital deduction The purpose of electing QTIP treatment for estate property is to assure qualification of that property for the estate tax marital deduction. A, B, D, and E are all nonsense options.

Jeremiah had a farm valued at $3.5 million which he left to a by-pass trust. His wife Marietta had property worth $2.3 million titled in her name. When Marietta died in 2012, she left her entire estate to her children. What was her estate tax liability?

$0 Marietta's estate was valued at $2,300,000 and her exemption was $5,120,000, so her estate would not have been taxed. The farm was in the name of the trust and passed directly to the children, without going through Marietta's estate or probate.

Laura and Jim acquired all of their property valued at $800,000 while married and living in N.J. They moved to Washington, acquired another $200,000 in property, and then divorced. For divorce settlement purposes how much of their property will be treated as community property?

$1 million $1 million since Washington recognizes a quasi-community property regime.

If a married individual in a common-law state gives their son a gift worth $20,000 and the requisite gift splitting election is made, for purposes of the gift tax computation, how much is that individual considered to have given?

$10,000 For purposes of the gift tax computation, that individual is considered to have given only $10,000. Their spouse is treated as giving the other $10,000, even if, in fact, none of the gift was her property.

In 2012, David makes a total gift of $1,000,000, of which $25,000 is a gift to a war veteran's association that is a qualified charitable organization. The remainder he divides equally among his daughters. What is the amount of his gift tax charitable deduction?

$12,000 The gift that qualifies for the charitable deduction is the $25,000 David donated to the war veteran's association. The annual exclusion applied is $13,000 for a charitable deduction of $12,000.

If one person has a one-half interest in a tenancy in common, how would a cash gift of $6,000 to the tenancy be treated?

$3,000 gift to that person If one person has a one-half interest in a tenancy in common, a cash gift of $6,000 to the tenancy would be treated as a $3,000 gift to that person. This would be added to other gifts made directly by the same donor to determine how much of the exclusion would be allowed.

When would individual ownership be appropriate?

- If an owner has a small estate - If an owner wants to retain ownership, but lets others enjoy and use the property - If owner wants absolute control over the property Individual ownership is appropriate if the owner wants absolute control over the property. It is also appropriate for those individuals with estates not large enough to be subject to estate tax.

Assuming that the donor was competent to make a gift, the donee was capable of accepting the gift, and there was a clear intention on the part of the donor to divest himself or herself of dominion and control over the gift property, what are the other requisites in gifting:

- A complete delivery to the donee of the gift - Acceptance of the gift by the donee - An irrevocable transfer of the present legal title Assuming that the donor was competent to make a gift, the donee was capable of accepting the gift, and there was a clear intention on the part of the donor to divest himself or herself of dominion and control over the gift property, three other elements must be present. An irrevocable transfer of the present legal title to the donee must be made so that the donor no longer has dominion and control over the property in question. The donor must make a complete delivery to the donee of the subject matter of the gift or the most effective way to command dominion and control of the gift. Another requirement is the acceptance of the gift by the donee. The measure of a gift is the difference between the value of the property transferred and the consideration received by the transferor. When the consideration received by the donor against a gift is equal to the value of the property transferred, it does not amount to a gift and amounts to the equivalent of a purchase.

Which of the following do not constitute an indirect gift?

- A new partner's services in business in exchange for a share of the partnership's earnings In cases where new partners are to contribute valuable services in exchange for their share of the partnership's earnings and where the business doesn't contain a significant amount of capital assets. A transfer to a corporation for inadequate consideration is a gift from the transferor to the corporation's other shareholders. Third-party transfers may be the medium for taxable gifts. In one case, an employee gave up his vested rights to employer contributions in a profit-sharing plan. If a person makes payments on an adult son's car or pays premiums on a life insurance policy his wife owns on his life, such payments are gifts.

Which of the following would not be included in an estate?

- Assets within an irrevocable trust Assets in an irrevocable trust should have been considered for gift taxes already. The grantor gave up control and ownership to any assets placed into an irrevocable trust during his or her lifetime.

A gratuitous transfer of property by a family-owned corporation to the father of the shareholders of a corporation could be treated as a gift from which of the following choices?

- Children to their father A gratuitous transfer of property by a family-owned corporation to the father of the shareholders of a corporation could be treated as a gift from the children to their father.

Under which property law is the property equally divided between the spouses as a result of death or in the case of divorce?

- Community property Community Property is any property acquired by the spouses in the course of their marriage. The property is divided equally between them if they divorce, or in the case of death of one of the partners.

Which of the following constitute grounds for the partitioning of a tenancy by the entirety?

- Consent of both parties - Divorce The tenancy by the entirety cannot be partitioned except with the consent of both parties or through divorce. A tenancy by the entirety cannot be severed by the act of a single spouse.

What can a will accomplish?

- Establish Testamentary Trust - Appoint Guardian - Appoint Executor - Indicate How Estate Should Be Distributed - Prevent will contests by including no-contest clause - Intentionally disinherit children in all states except Louisiana - Transfer property interests to partners

Which of the following financial planning components can help with the transferring of Cecilia's property held as single ownership in the event that she passes away?

- Estate Planning Estate planning will help Cecilia make the right decisions about her property ownership in preparing for the transfer of her assets upon death. The other components of financial planning will support other financial decisions.

Maureen is studying up on property titling as part of planning for her estate. Which of the following statements are true about the destination of her assets in the event of her death?

- Her summer home left to her alone from her mother would go through her probate estate. - Her mutual fund accounts held as joint tenant accounts with her children will go directly to them. Maureen's house held in tenants by entirety would be left to her widower. Her summer home she owned by herself and her winter cabin held in tenants in common will become part of her estate. Finally, her mutual fund account interests would go to the surviving joint tenants.

Jennifer, Michelle, Tom and Heather own a summer home as joint tenants. Tom passes away. Who will receive Tom's interest in the summer home? Click all that apply.

- Jennifer - Michelle - Heather Joint tenant ownership comes with the right of survivorship. This means that in the event that one of the owners passes away, the other surviving owners of the property would receive the decedent's interest. In this case, Jennifer, Heather and Michelle would receive Tom's JT interest in property that was included in his estate.

Which of the following allows the grantor to retain control and use of his or her assets?

- Joint Tenancy - Revocable Trust - Contracts Joint tenancy means that the owner's interest is not passed to the surviving owner until his or her death. Funds of a revocable trust are still controlled and used by the grantor. Use of contracts such as IRAs to pass assets to beneficiaries does not give any interest of the assets to the beneficiaries until the owner passes away.

Why is a lifetime gifting program beneficial for tax planning purposes?

- Lowers the size of the estate - Lowers the amount of estate taxes Gifting estate assets during lifetime reduces the amount of assets which will be included in the decedent's estate after death. It is not advantageous for a donee to receive the donor's basis in property, since a future sale might result in a capital gain. Property that passes to a beneficiary upon the owner's death receives a step-up in basis to fair market value.

The donor's spouse must be?

- Must be a US citizen at the time the gift is made, or the marital deduction is denied. Gifts to a resident alien spouse qualify for a $139,000 annual exclusion.

The property of transferred to the donee-spouse must not be?

- Must not be the type of terminable interest that will disqualify the gift for the marital deduction.

If a mother and daughter are in litigation and the daughter is claiming a large sum of money, a compromise payment by the mother to the daughter would be considered?

- Not a gift A compromise payment by the mother to the daughter is not a gift. However, in an interfamily situation in which the court is not convinced that a bona fide arm's length adversary proceeding was present, the gift tax will be imposed

What is a grantor?

- One who establishes and funds the trust - An individual who conveys or transfers ownership of property

Peter and his wife titled their home as JTWROS. When his wife died the property avoided probate. Peter as sole owner wants to avoid probate again and continue to live in his home for life. Therefore he changed his deed to create a life estate in the home, and gave his son the remainder interest. What are the gift tax consequences of this transaction?

- Peter has made a gift to his son for the present value of the remainder interest in property. When Peter changed the deed, he made a gift to his son of the present value of the home's remainder interest. This is less than the home's current fair market value. The gift of the home's remainder interest is a future interest gift, so annual exclusions do not apply. Annual exclusions reduce the taxable amount of present interest gifts.

Carol, Ann, and Barbara hold one-half, one-third, and one-sixth interests respectively in a farm. They all share an undivided interest in the farm. What type of ownership applies to this situation?

- Tenancy in Common Since each owner shares an undivided interest, even though each owns an unequal share, this is considered a tenancy in common. In a joint tenancy each owner has a simultaneous, equal share of the entire property. It is not single ownership because there is more than one owner. It is not tenancy by the entirety because the property is not owned by husband and wife.

A co-ownership discount is a feature of:

- Tenancy in Common When the different tenants in Tenancy in Common, have conflicting interests regarding the sale of the common property and do not agree to the sale, the property gets a discount in valuation. This is referred to as a co-ownership discount. This discount is not available in the case of Sole Ownership, Joint Tenancy with Right of Survivorship and Tenancy by the Entirety.

Lifetime control and postmortem control of property interests are characteristics of property held in:

- Tenancy in Common - Sole ownership The most important feature of sole ownership and ownership of property titled Tenancy in Common is that the individual has total control over their property interests both during lifetime and upon death.

Graydon and Leisa recently got married and are about to purchase a new home. Which of the following property titles will cause the house to have to go through probate in order to be transferred to the surviving spouse?

- Testamentary Q-TIP - Tenants in Common Although the house would count as part of the estate if it was held within a living trust, it would not need to go through probate to pass to the other. Right of survivorship will pass the home to the surviving owner without going through probate. Testamentary QTIPs are trusts which are activated after probate. Tenants in common owners' interest goes through probate upon their death rather than automatically to surviving owners.

Tom's will gave his wife Mary a life estate in his Italian villa at his death in 2012. His executor did not elect to use the Q-TIP election. What are the estate tax consequences for Tom's estate?

- The FMV of the villa is included in Tom's estate but the estate tax marital deduction is not available to offset the estate tax liability. - The FMV of the villa is included in Tom's estate but his applicable credit amount is available to offset any estate tax liability. Tom has given Mary terminable interest property that does not receive a marital deduction in his estate, to reduce the value of his taxable estate in 2012. Tom can use his applicable credit amount to reduce any estate tax liability. Mary does not automatically receive a general power of appointment over the property since it must be given to her expressly in Tom's will. If the executor had made a Q-TIP election, the marital deduction would have been available to Tom's estate.

After a divorce, Cecilia opened an account for herself at a local bank under her own name. She became afraid that her ex-husband would be able to access her account without her approval. Which of the following points would you make to Cecilia to clarify how her single account works?

- The bank should not perform any transactions without Cecilia's approval - Someone whom she granted permission to through power of attorney can make transactions from her account One benefit to owning assets as a single owner is the ability to maintain sole control over the assets. Cecilia is the only person who can request transactions for the account. If she grants the power of attorney to someone, that person will have the authority to make transaction requests on her behalf.

What happens to an estate if a will is declared invalid?

- The estate is divided in accordance with laws of intestacy. If a will was declared invalid, the estate will be distributed by the courts under the state's laws of intestacy. A will may be determined to be invalid if its execution did not follow the guidelines under state law, or was improperly witnessed or signed.

In community property states, if one spouse makes a gift of community property to a third person without the consent of the other spouse, the gift is ordinarily voidable rather than void. Which of the following statements are true regarding such gifts?

- The gift can be voided only at the request of the nondonor spouse. - If the spouses are still married, the entire gift is returned to the community estate. - If the community has been terminated, the nondonor spouse has a right to recapture one-half of the gift. The gift can be voided only at the request of the nondonor spouse. If the spouses are still married, and the non-donor spouse requests that the gift be voided, the entire gift is returned to the community estate. However, if the community has been terminated, for example, by divorce or death, the nondonor spouse has the right to recapture only one-half of the gift. The other half is allowed to remain with the donee. A marital deduction is available for the entire amount of the gift, where one spouse gives his or her community interest in property to the other.

A mother re-titles the deed to her home as JTWROS with her son. What are the consequences of this action?

- The mother made a gift of ½ of her home to her son. - When the mother dies, 100% of the FMV of the home will be included in her estate, but she can use her unified credit amount to offset up to $5,120,000 in value, in 2012. - When the mother dies, her executor cannot sell the house to pay her estate taxes but a unified credit of 1,772,800 is available to offset up to $5,120,000 in estate taxes. The mother made a gift of ½ of the FMV of her house to her son when she gave him ownership of the property. If she is sued, creditors can place a lien on the home and reach her son's interest. If she becomes incompetent, her son cannot sell her JT interest in the home without obtaining court approval. When the mother dies, the house passes automatically to her son and is not available to her estate. The full FMV of the home will be included in her estate but is offset by her unified credit amount. The son will receive a full step-up in basis at her death.

A disadvantage of tenancy by the entirety property is that it may "over-qualify" the estate for the marital deduction. State True or False.

- True The decedent spouse includes ½ of the FMV of the tenancy by the entirety property in his gross estate which is subject to the marital deduction. This means the decedent's applicable credit is not used to offset the estate tax liability, and the decedent's estate is "over-qualified" for the marital deduction. It is tempting to "over-utilize" the marital deduction at the first death. Remember, everyone has an applicable exclusion amount from the federal estate tax. So, instead of leaving this applicable exclusio n amount outright to the spou se, where it will be taxed at the second death, why not put it in a trust that will provide an income to the surviving spouse but bypass the second estate at death? Then the survivor can use his or her own applicable exclusion amount to shelter other assets from tax at the second death. This is accomplished through the use of a "bypass trust."

The reasons for gifting are either to save tax or to see the donee benefit by the gift. Giving away assets more than three years prior to death, other than closely held stock will make it easier to qualify for:

- an IRC Section 303 redemption of stock - an IRC Section 6166 installment payout of taxes attributable to a closely held business interest - an IRC Section 2032A special use valuation for certain real property used for farming or closely held business purposes Giving away assets other than closely held stock will make it easier to qualify for an IRC Section 303 redemption of stock, an IRC Section 6166 installment payout of taxes attributable to a closely held business interest, and an IRC Section 2032A special use valuation for certain real property used for farming or closely held business purposes.

What are the three nontax advantages of lifetime gifting?

- watching the donee enjoy the gifted property - allowing the donor to witness how a donee utilizes the gifted asset - making the donor feel good about being able to make a gift

If the spouses are still married, the entire gift is returned to the community estate. However, if the community has been terminated by divorce or death, what percentages of the gift will the spouse will have the right to recapture?

50 percent The spouse has the right to recapture only one-half of the gift. The other half is allowed to remain with the donee. Therefore, it is advisable to obtain both spouses' consent prior to the lifetime transfer of community assets.

Suppose the husband and wife reside in New York State. Under New York State law, the husband's earnings are considered his separate property. The couple retires and moves to California, which is a quasi-community property state. If the husband dies before the wife, what percentage of the husband's estate goes to the wife?

50% At least one-half of these assets, which are now considered quasi-community property at his death, must go to the wife. If the wife dies first, she cannot dispose of any of these assets because they belong to the husband until he dies.

Assuming that the gift is complete, it can be subject to gift tax, if the donee is:

Almost any party can be the donee or recipient of a gift subject to tax. The donee can be an individual, partnership, corporation, foundation, trust or other person. In fact, a gift can be subject to the tax, assuming the gift is complete even if the identity of the donee is not known at the date of the transfer and cannot be ascertained.

If a grandmother purchases a U.S. savings bond that is registered as payable to her and to her two children as co-owners, no gift is made to the grandchildren until one of them ___________ the bond for cash.

No gift is made to the grandchildren until one of them surrenders the bond for cash. Federal rather than state law governs transfer of U.S. government bonds. Even if state law requirements for a valid gift are met, for tax purposes no completed gift has been made until the registration is changed in accordance with federal regulations.

If Sean left 50% of his estate to his spouse Karen and 50% to charity, how much of it would be deductible in 2012?

There is an unlimited marital deduction- Marital and charitable transfers are 100% deductible.


संबंधित स्टडी सेट्स

Study Sync First Read: Speech to the Second Virginia Convention (comprehension questions)

View Set

Timbre - The Character of a Sound (Chapter 6)

View Set

Consumer Behavior (Ch. 8-11 Quizzes)

View Set

SCOM372/4 Ch. 14 Transportation MCQ

View Set

Nursing Assessment: Immune Function

View Set

Communication 101 Final Study Guide (Quizzes)

View Set